Download as pdf or txt
Download as pdf or txt
You are on page 1of 50

CHAPTER

05
Limits
Learning Part
Session 1
● Definition of Limits

● Indeterminate Forms

● L’Hospital’s Rule

● Evaluation of Limits

Session 2
● Trigonometric Limits

Session 3
● Logarithmic Limits

● Exponential Limits

Session 4
● Miscellaneous Forms

Session 5
● Left Hand and Right Hand Limits

Session 6
● Use of Standard Theorems/Results

● Use of Newton-Leibnitz’s Formula in Evaluating the Limits

● Summation of Series Using Definite Integral as the Limit

Practice Part
● JEE Type Examples
● Chapter Exercises

Arihant on Your Mobile !


Exercises with this #L
symbol can be practised on your mobile. See title inside to activate for free.
Session 1
Definition of Limits, Indeterminate Forms,
L’Hospital’s Rule, Evaluation of Limits

Definition of Limits (b) When the left tendency is not the same as its
right tendency Here, left tendency is l1 and right
Let lim f ( x ) = l. It would mean that when we approach
x ®a tendency is l 2 , clearly left tendency (l 1 ) is not same as
the point x = a from the values which are just greater than right tendency (l 2 ). In this case, we say that the limit of
or smaller than x = a, f ( x ) would have a tendency to move f ( x ) at x = a will not exist.
closer to the value ‘l ’.
i.e. lim f ( x ) = Doesn’t exist.
This is same as saying ‘difference between f ( x ) and l can x ®a

be made as small as we feel like by suitably choosing x in


Y
the neighbourhood of x = a’.
Mathematically, we write this, as lim f ( x ) = l, which is y = l2
x ®a
equivalent of saying that, | f ( x ) – l | < e , " x whenever y = l1
0 < | x - a | < d and e and d sufficiently small +ve
numbers.
X
O x=a
It is clear from the above discussion that, if we are x=a+h
interested in finding the limit of f ( x ) at x = a, the first x=a–h
thing we have to make sure that f ( x ) is well defined in
the neighbourhood of x = a and not necessarily at x = a Figure 5.2
(that means x = a may or may not be in the domain of
f ( x )), because we have to examine its behaviour or (c) When the left tendency and/or right tendency is
tendency in the neighbourhood of x = a. not fixed As shown in the figure 5.3, it is clear that
in this case, the function has erratic behaviour in the
Following possibilities may arise : neighbourhood of x = a and it will not be possible to
(a) Left tendency is same as its right tendency As talk about the left and right tendencies of the function
shown in figure 5.1, when we approach x = a from in the neighbourhood of x = a.
the values which are just less than a, f ( x ) has a In this case, we conclude that the limit of f ( x ) at
tendency to move towards the value l (left tendency). x = a will not exist.
Similarly, when we approach x = a from the values
i.e. lim f ( x ) = Doesn’t exist.
which are just greater than a, f ( x ) has a tendency to x ®a
move towards the value l (right tendency). Y
In this case, we say f ( x ) has limit l at x = a,
i.e. lim f ( x ) = l .
x ®a
Y y = f(x)
y = f(x)

y=l

X x a
O
x=a
Figure 5.3
Figure 5.1
Chap 05 Limits 247

Remarks Infinity ( ¥) is a symbol and not a number. It is a symbol


1. Normally, students have the Y for the behaviour of a variable which continuously
perception that limit should be a increases and passes through all limits. Thus, the
finite number. But, it is not always
so. It is quite possible that f ( x ) has
statement x = ¥ is meaningless, we should write x ® ¥.
infinite limit at x = a. Similarly, – ¥ is a symbol for the behaviour of a variable
If xlim
® a
f ( x ) = ¥, it would simply O X which continuously decreases and passes through all
mean that function has tendency to limits. Thus, the statement x = – ¥ is meaningless, we
assume very large positive values in Figure 5.4
neighbourhood of x = a (as shown in
should write x ® – ¥.
Y
figure 5.4). 1 1
1 Also, ® 0, if x ® + ¥ and ® 0, if x ® – ¥.
For example lim =¥ O
X x x
x ® 0 | x|

which indicates the left tendency as We cannot plot ¥ on paper. Infinity does not obey laws of
well as right tendency are the same. elementary algebra.
Again, if lim f ( x ) = - ¥, it would (i) ¥ + ¥ = ¥ is indeterminate
x ® a
simply mean that the function has Figure 5.5
tendency to assume very large
(ii) ¥ – ¥ is indeterminate.
negative values in the neighbourhood of x = a as shown in
figure 5.5.

For example (as shown in figure 5.5) lim


–1
=– ¥
L’Hospital’s Rule
x ® 0 |x|
f (x ) 0 ¥
At the end we discuss the case when left tendency is (– ¥) and This rule states that, if lim , reduces to or ×
right tendency is ( + ¥) (i.e. f ( x ) does not have unique
x ®a g( x ) 0 ¥
tendency). Then, differentiate numerator and denominator till this
Thus, in this case limit does not exist. form is removed.
1 Y
For example lim does not exist, f (x ) f ¢ (x )
x ® 0 x
i.e. lim = lim , provided the later limit exists.
since left tendency is (– ¥) and right x ® a g( x ) x ® a g ¢ (x )
tendency is ( + ¥) as shown in X
figure 5.6. O æ 0 ¥ö
But, if it again take form ç or ÷ ,
2. If f ( x ) is well defined at x = a, it è 0 ¥ø
doesn’t imply that Figure 5.6 f (x ) f ¢ (x ) f ¢¢( x )
lim f ( x ) = f ( a). Because, it is quite then lim = lim = lim
x ® a
possible that f ( x ) is well
x ®a g ( x ) x ® a g ¢ ( x ) x ® a g ¢¢( x )
defined at x = a but not in the neighbourhood of x = a or f ( x ) is æ0 ¥ö
well defined in the neighbourhood of x = a, but doesn’t have a and this process is continued till ç or ÷ form is removed.
è0 ¥ø
unique tendency.

Remark
Indeterminate Forms L’Hospital’s rule is applicable to only two indeterminate
¥
forms æç or ö÷ .
0
If direct substitution of x = a while evaluating lim ( x ) è0 ¥ø
x ®a
leads to one of the following forms
0 ¥ x 6 – 24 x – 16
, , ¥ – ¥, 1 ¥ , 0 0 , ¥ 0 , ¥ ´ 0 then it is called e.g. Evaluate lim ×
0 ¥ x ®2 x 3 + 2x – 12
indeterminate form. x 6 – 24 x – 16 é0 ù
Sol. We have, lim êë 0 formúû
x2 –1 0 x ®2 x + 2x – 12
3
e.g. lim = indeterminate form.
x ®1 x – 1 0 6x 5 – 24
= lim [by L’Hospital’s rule]
x n – an 0 x ®2 3x 2 + 2
lim = indeterminate form.
x ®a x – a 0 6 (2)5 – 24 168
= = = 12
sin x 0 3 ( 2) + 2
2
14
lim = indeterminate form.
x ®0 x 0
248 Textbook of Differential Calculus

é 23 x 3 25 x 5 ù é x3 x5 ù
Frequently Used Series Expansions x ê2x +
3
+2
15
+...ú –2x ê x +
3
+2
15
+...ú

1. ex = 1 +
x
+
x2 x3
+ + ... = lim ë û ë û
1! 2 ! 3!
x ®0 (2 sin 2 x )2

x × log a (log a) 2 x 2 æ8 2ö æ 64 4ö
2. ax = 1 + + + ... x 4 ç - ÷ + x 6 ç - ÷ + ...
1! 2! è3 3ø è 15 15 ø
= lim
[where, a Î R+ ] x ®0
æ x3 x5 ö
4

nx n ( n – 1) x 2 4 çx - + – ...÷
3. ( 1 + x ) n = 1 + + è 3! 5! ø
1! 2!
2 + 4 x 2 + ... 2 1
+
n ( n – 1) ( n – 2) x 3
+ ... n Î R and|x| < 1
= lim 4
= =
x ®0 4 2
3! æ x 2 ö
4 ç1 - + ...÷
4. log ( 1 + x ) = x –
x 2
+
x

x 3
+ ...
4
è 3 ! ø
2 3 4
[where –1 £ x £ 1]

5.
x n – an
x–a
= x n – 1 + x n – 2a + x n – 3a2 + ... + an – 1 Evaluation of Limits
Now, according to our plan, first of all we shall learn the
æ x 11x 2 ö
6. ( 1 + x )1 /x = e çç1 – + + ...÷÷ evaluation of limits of different forms and then learn the
è 2 24 ø existence of limits.
x 3 x5 x 7 There are eight indeterminate or meaningless forms,
7. sin x = x – + – + ...
3! 5 ! 7! which are
x 2 x4 x6 0 ¥
8. cos x = 1 – + – + ... (i) (ii) (iii) ¥ – ¥ (iv) ¥ ´ ¥
2! 4 ! 6 ! 0 ¥
x3 2 5 17 7 (v) ¥ × 0 (vi) 0 0 (vii) ¥ 0 (viii) 1 ¥
9. tan x = x + + x + x + ...
3 15 315 We will divide the problems of evaluation of limits in five
12 3 12 × 32 5 12 × 32 × 52 7 categories, which are
10. sin–1 x = x + x + x + x + ...
3! 5! 7! 1. Limit of algebraic functions
x 3 x5 2. Trigonometric limits
11. tan–1 x = x – + + ...
3 5
3. Logarithmic limits
x2 5x 4 61x 6
12. sec –1 x = 1 + + + + ... 4. Exponential limits
2! 4! 6!
5. Miscellaneous forms
2 2 2 × 22 4 2 × 22 × 4 2 6
13. (sin–1 x ) 2 = x + x + x + ... Now, we discuss one by one in details.
2! 4! 6!
x 3 x 4 2x 6
14. x cot x = 1 –
3
+ –
45 945
+ ...
Limit of Algebric Functions
2 4 6
15. sec x = 1 +
x
+
5x
+
61x
+ ...
In this section, we evaluate limit of algebraic functions
2 24 720 when variable tends to a finite or infinite value. While
x2 7x4 31x 6 0 ¥
16. x cosec x = 1 + + + + ... evaluating algebraic limits the form , and ¥ - ¥ arise,
6 360 15120 0 ¥
which we will discussed here.

x tan 2x – 2x tan x 0
y Example 1 Evaluate lim 2
× (i) Form
x ®0 (1 – cos 2x ) 0
[IIT JEE 1999] This form can be resolved by factorisation method,
x tan 2x – 2x tan x rationalisation method or by using the formula
Sol. We have, lim
x ®0 (1 – cos 2x )2 x n - an
lim = na n - 1 , which are discussed below.
x ®a x - a
Chap 05 Limits 249

(a) Factorisation Method x+h– x


y Example 4 Evaluate lim .
In this method, numerators and denominators are h ®0 h
factorised. The common factors are cancelled and the rest x +h – x
output is the result. Sol. Method I We have, lim
h ®0 h
x 2 - 3x + 2 x +h – x x +h + x é0 ù
y Example 2 Evaluate lim × = lim ´ êë 0 form úû
x ®1 x –1
h ®0 h x +h + x

x 2 – 3x + 2 ( x + h )–( x ) h
é0 ù = lim = lim
Sol. Method I We have, lim êë 0 form úû h ®0 h( x + h + x ) h ® 0 h( x + h + x )
x ®1 x –1
( x – 1)( x – 2) 1 1
= lim = lim =
x ®1 ( x – 1) h ®0 x +h + x 2 x
[as x 2 - 3x + 2 = ( x - 1)( x - 2)] Method II (L’Hospital’s rule) We have,
= lim ( x – 2) [as x –1 ¹ 0] x +h – x é0 ù
x ®1
L = lim êë 0 form úû
h ®0 h
= 1–2 = - 1 \ Applying L’Hospital’s rule,
x 2 – 3x + 2 é0 ù 1
Method II We have, L = lim êë 0 form úû –0
x ®1 x –1 2 x +h
L = lim
So, applying L’Hospital’s rule, h ®0 1
2x – 3 2 – 3 [differentiating numerator and denominator w.r.t. h]
L = lim = =–1
x ®1 1 1 1
=
[i.e. differentiating numerator and 2 x
denominator separately]
x - 2a + x - 2a
y Example 5 Evaluate lim .
x 3 – x 2 log x + log x – 1 x ® 2a
y Example 3 Evaluate lim . x 2 - 4a 2
x ®1 x 2 –1 x - 2a + x - 2a é0 ù
Sol. We have, lim êë 0 form úû
x 3 – x 2 log x + log x – 1 x ® 2a x - 4a
2 2
Sol. We have, lim
x ®1 x2 –1 x - 2a
x - 2a +
( x 3 – 1)–( x 2 – 1) log x é0 ù x + 2a
= lim = lim
x ®1 ( x 2 – 1) êë 0 form úû x ® 2a x - 2a x + 2a
1 x - 2a
( x –1){ x 2 + x + 1 – ( x + 1) log x } = lim +
= lim x ® 2a x + 2a x + 2a ( x + 2a )
x ®1 ( x –1)( x + 1)
éQ x 3 - 1 = ( x - 1) ( x 2 + x + 1),ù 1 1
= =
ê ú 4a 2 a
êë x - 1 = ( x - 1) ( x + 1)
2
úû
x 2 + x + 1 – ( x + 1) log x (c) Based on Standard Formula
= lim
x ®1 ( x + 1) x n – an
lim = na n – 1 , where n is a rational number.
12 + 1 + 1 – (1 + 1) log 1 3 x ®a x –a
= = [as log 1 = 0]
1+1 2 x n – an
Proof Let f ( x ) =
x –a
(b) Rationalisation Method
= x n – 1 + ax n – 2 + a 2 x n – 3 + ...+ a n – 1
Rationalisation is followed when we have fractional
1 1 \ lim f ( x ) = lim ( x n – 1 + ax n – 2 + a 2 x n – 3 + ...+ a n – 1 )
x ®a x ®a
powers (like , etc.) on expressions in numerator or
2 3 =a + a × a n – 2 + a 2 × a n – 3 + ...+ a n – 1
n–1
denominator or in both. After rationalisation the terms are 1444444 424444444 3
factorised which on cancellation gives the result. n terms
= a n – 1 + a n – 1 + a n – 1 + K upto n terms = n × a n – 1
250 Textbook of Differential Calculus

f (x) = ax, when 0<a<1 f (x) = ax, when a = 1


x 3 – 23 Y Y
y Example 6 Evaluate lim .
x ®2 x – 2
1
x 3 – 23 é0 ù X
Sol. We have, lim êë 0 formúû O
X
O
x ®2 x –2

é x n – an ù
= 3 ( 2) 3 – 1 ê x ®a
Q lim = n × an – 1 ú
ë x –a û Figure 5.7 Figure 5.8
= 3 (2) = 12
2
f(x) = ax,when a>1
Y
x + x 2 + ... + x n - n
y Example 7 Evaluate lim .
x ®1 x -1
X
x + x 2 + x 3 + ... + x n - n O
Sol. We have, lim
x ®1 x -1
Figure 5.9
( x - 1) + ( x 2 - 12 ) + ( x 3 - 13 )+...+( x n - 1n )
= lim
x ®1 ( x - 1) Now, see the graph for a x , when a > 1 . This graph appears
ì x - 1 x 2 - 12 x 3 - 13 x n - 1n ü to touch X-axis in the negative side of X-axis and
= lim í + + +...+ ý
x ®1 x - 1 x -1 x -1 x -1 þ thereafter it increases rapidly. This is why because
î
lim a x ® 0, again you will also find the result,
= 1 + 2 (1)2 -1 + 3 (1)3 -1 + ... + n (1)n -1 x ®-¥
lim a x ® ¥
= 1 + 2 + 3 + ... + n x ®¥

=
n ( n + 1) ì¥, if a > 1
ï
2 Thus, we have lim a = í1, if a = 1
x
x ®¥
3 ï0, if 0 £ a < 1
y î
y Example 8 The value of lim as
x ®1 x3 - y 2 -1 This type of problems are solved by taking the highest
y ®0
power of the terms tending to infinity as common from
( x , y ) ® (1, 0) along the line y = x - 1 is numerator and denominator. That is after they are
(a) 1 cancelled and the rest output is the result (or apply
(b) -1 L’Hospital’s rule).
(c) 0 x2 + 5
(d) Doesn’t exist y Example 9 Evaluate lim .
x ®¥ x 2 + 4x + 3
Sol. As, y ® x - 1 or x ® y + 1 x2 + 5
Sol. Let L = lim
(y )3 x ®¥ x + 4x + 3
2
\ lim
y ®0 ( y + 1) - y - 1
3 2
Dividing numerator and denominator by x 2 , we get
5
Using L, Hospital’s rule, 1+ 2
x 1+0
3y 2 6y L = lim = =1
lim = lim x ®¥ 4
1+ + 2
3 1 + 0 + 0
y ®0 3 (y + 1) - 2y
2 y ®0 6 ( y + 1) - 2 x x
0 K
=0 = [because ® 0, when x ® ¥, where K is any constant]
6 x
Hence, (c) is the correct answer. x2 + 5 é¥ ù
Aliter We have, L = lim êë ¥ form úû
x ®¥ x + 4x + 3
2
(ii) Algebraic Function of ¥ Type
2x é¥ ù
¥ Applying L’Hospital’s rule, L = lim
® ¥ 2x + 4 êë ¥ form úû
(a) Form x

¥ 2
Again, applying L’Hospital’s rule, L = lim =1
First we should know the limiting values of a x (a > 0 ) as x ® ¥2

x ® ¥. See the graphs of this function.


Chap 05 Limits 251

(n + 2)! + (n + 1)! An Important Result


y Example 10 Evaluate lim .
n ®¥ (n + 2)! – (n + 1)! If m, n are positive integers and a0, b0 ¹ 0 and non-zero real
(n + 2)(n + 1) ! + (n + 1) ! numbers, then
Sol. We have, lim a x m + a1x m – 1 + K + am – 1x + am
n ®¥ (n + 2)(n + 1) ! – (n + 1) ! lim 0 n
x ® ¥ b x + b xn – 1 + K + b
n – 1x + bn
(n + 1) ![ n + 2 + 1] ( n + 3) é¥ ù 0 1
= lim = lim êë ¥ form úû ì 0, m<n
n ®¥ (n + 1) ![ n + 2 – 1] n ® ¥ (n + 1) ï a
1 + 3/n 1 + 0 é 1 ù ïï 0 , m=n
= lim = =1 êëQ n ® 0, as n ® ¥ úû
= í b0
n ® ¥ 1 + 1/n 1+0 ï ¥, m > n when a0b0 > 0
ï
ïî– ¥, m > n when a0b0 < 0
(b) ¥ - ¥ Form
Such problems are simplified (generally rationalised) first, ax 2 + b
y Example 13 Evaluate lim , when a ³ 0.
æ ¥ö x ®¥ x + 1
thereafter they generally acquire ç ÷ form.
è ¥ø
ax 2 + b
Sol. Here, if a ¹ 0 ; lim
y Example 11 Evaluate lim ( x – x 2 + x ) . x ®¥ x + 1
x ®¥ [as degree of numerator > degree of denominator]
Sol. We have, lim ( x – x 2 + x ) ax 2 + b
x ®¥ = lim =¥ [as a > 0]
x ®¥ x + 1

x – x2 + x x + x2 + x 0× x 2 + b
= lim ´ [ ¥ – ¥ form] Again, if a = 0 ; lim = lim
b
=0
x ®¥ 1 x + x2 + x x ®¥ x +1 x ®¥ x + 1

x 2 – (x 2 + x ) –x [as degree of numerator < degree of denominator]


= lim = lim ax 2 + b ì ¥, a > 0
x ®¥
x+ x +x 2 x ®¥
x + x2 + x \ lim =í
x ®¥ x + 1 î0, a = 0
–x
= lim æx2 +1 ö
x ®¥ ì 1ü y Example 14 If lim ç – ax – b ÷ = 0, find the
x í1 + 1 + ý x ®¥ è x + 1
î xþ ø
values of a and b.
–1 1 é 1 ù æx2 + 1 ö
= lim =– êëQ x ® 0, as x ® ¥ úû
x ®¥ 1 2 Sol. Given, lim ç – ax – b ÷ = 0
1+ 1+ x ®¥ è x + 1 ø
x
x 2 + 1 – ax 2 – ax – bx – b
Þ lim =0
y Example 12 Evaluate lim ( x + x + 1 – x + 1 ). 2 2
x ®¥ x +1
x ®¥
x 2 (1 – a ) – x (a + b ) + (1 – b )
Sol. We have, lim ( x 2 + x + 1 – x 2 + 1 ) [ ¥ – ¥ form ] Þ lim =0
x ®¥
x ®¥ x +1
Since, the limit of above expression is zero.
( x 2 + x + 1 – x 2 + 1) ( x 2 + x + 1 + x 2 + 1)
= lim ´ \ Degree of numerator < Degree of denominator.
x ®¥ 1 ( x 2 + x + 1 + x 2 + 1) So, numerator must be a constant, i.e. a zero degree
polynomial.
( x 2 + x + 1) – ( x 2 + 1)
= lim \ 1 – a = 0 and a + b = 0
x ®¥ x2 + x + 1 + x2 + 1 Hence, a = 1 and b = –1
x æx2 +1 ö
= lim y Example 15 If lim ç – ax – b ÷ = 2, find the
x ®¥ x2 + x + 1 + x2 + 1 x ®¥ è x + 1 ø
1 1 1 values of a and b.
= lim = =
x ®¥ 1 1 1 1+1 2 æx2 + 1 ö
1+ + 2 + 1+ 2 Sol. We have, lim ç – ax – b ÷
x x x x ®¥ è x + 1 ø
é 1 ù x 2 (1 – a ) – x (a + b ) + (1 - b )
= lim =2
êëQ x ® 0, as x ® ¥ úû x ®¥ ( x + 1)
252 Textbook of Differential Calculus

Since, limit of above expression is a finite non-zero number. n ( n + 1)


Sol. As, Sn =
\ Degree of numerator = Degree of denominator 2
Þ 1–a = 0 Þ a = 1 n ( n + 1) n 2 + n - 2 ( n + 2) ( n - 1)
Þ Sn - 1 = -1= =
Putting a = 1 in above limit, we get 2 2 2
- x (1 + b) + (1 + b ) Sn æ n ö æn + 1ö
lim =2 \ =ç ÷ç ÷
x ®¥ x +1 Sn - 1 è n - 1 ø è n + 2 ø
Þ – (1 + b ) = 2 Þ b = –3
æ2 3 4 n öæ3 4 5 n + 1ö
Hence, a = 1 and b = - 3. Þ Pn = ç × × K ÷ç × × K ÷
è1 2 3 n - 1ø è 4 5 6 n + 2ø
æx2 +1 ö
y Example 16 If lim ç – ax – b ÷ = ¥, find a and æn ö æ 3 ö
x ®¥ è x + 1
=ç ÷ç ÷
ø è 1 ø èn + 2ø
b. 3n
\ lim Pn = lim =3
n ®¥ n ® ¥n + 2
æx2 + 1 ö
Sol. Given, lim ç – ax – b ÷ = ¥
x ®¥ è x + 1 ø 1
y Example 18 If lim -1
= 1, x lies in the
Þ lim
x (1 – a ) – x (a + b ) + (1 – b )
2

n ®¥ (sin x )n + 1
x ®¥ x +1 interval
The limit of above expression is infinity. (a) ( - sin 1, sin 1) (b) ( - 1, 1)
\ Degree of numerator > Degree of denominator (c) (0, 1) (d) ( - 1, 0)
Þ 1–a > 0 Þa ¹ 1 1
Sol. Here, lim -1
= 1 is possible only, if
Hence, a < 1 and b can assume any real value. n ®¥ (sin x )n + 1

y Example 17 Let S n = 1 + 2 + 3 + L + n -1 < sin -1 x < 1

S2 S S S Þ x Î ( - sin 1, sin 1)
and Pn = × 3 × 4 K n , Hence, (a) is the correct answer.
S 2 - 1 S 3 - 1 S 4 - 1 Sn - 1
where n Î N (n ³ 2). Find lim Pn .
n ®¥

Exercise for Session 1


g( x ) f (a ) - g(a ) f ( x )
1. If f (a ) = 2, f ¢ (a ) = 1, g(a ) = - 1, g ¢ (a ) = - 2, then lim is
x ®a x -a
(a) - 5 (b) 3 (c) - 3 (d) 5
x cos x - log (1 + x )
2. The value of lim is
x ®0 x2
1 1
(a) 1 (b) (c) (d) None of these
4 2
2
e x - cos x
3. The value of lim is
x ®0 x2
3 3 1 1
(a) (b) - (c) (d) -
2 2 2 2
cos x - cos a
4. The value of lim is
x ®a cot x - cot a
(a) - sin3 a (b) cos3 a (c) sin3 a (d) cot a
Chap 05 Limits 253

æ 1 ö
5. The value of lim ç 2 - cot x ÷ is
x ®0 èx ø
(a) 0 (b) 1
1
(c) (d) None of these
4

6. The value of lim ( a 2x 2 + ax + 1 - a 2x 2 + 1), (a > 0) is


x ®¥
1 1
(a) (b) -
2 2
(c) Doesn’t exist (d) None of these
13 + 23 + 33 + . . . + n 3
7. The value of lim is
n®¥ (n 2 + 1)2
1 1
(a) (b)
4 2
1
(c) (d) None of these
2 2
1× n + 2 × (n - 1) + 3 × (n - 2) + . . . + n × 1
8. The value of lim is
n®¥ 12 + 22 + . . . + n 2
(a) 1 (b) -1
1 1
(c) (d)
2 2
an + b n
9. The value of lim , (where a > b > 1) is
n ® ¥ an - b n

(a) 1 (b) -1
1 1
(c) (d)
2 2
Session 2
Trigonometric Limits
Trigonometric Limits y Example 21 Evaluate lim 2 –x sin (2 x ).
x ®¥
To evaluate trigonometric limits the following results are
1
given below Sol. Since, 2–x = . We know that, as x ® ¥, 2x ® ¥
2x
sin x tan x
(i) lim =1 (ii) lim =1 \ The given limit = 0 ´
x ®0 x x ®0 x
[A finite number between –1 and +1] =0
sin–1 x tan–1 x sin (2x )
(iii) lim =1 (iv) lim =1 Hence, lim =0
x ®0 x x ®0 x x ®¥ ( 2x )
sin x 0 p
(v) lim = (vi) lim cos x = 1 y Example 22 Evaluate lim e x sin (d /e x ).
x ®0 x 180° x ®0
x ®¥
sin ( x – a ) tan ( x – a )
(vii) lim = 1 (viii) lim =1 Sol. When x ® ¥, e ® ¥ x
x ®a x –a x ®a x –a d finite
But, angle of sine = x = =0
1 – cos x e ¥
y Example 19 Evaluate lim . sin (d/e x ) sin d/e x
x ®0 x2 \ The given limit = lim = lim ´d
1–cos x
x ®¥ 1/e x d
® 0 d/e
x

Sol. We have, lim ex


x ®0 x2 =1 ´ d = d
2 sin 2 x/ 2 2 sin 2 x/ 2 é0 ù
= lim = lim êë 0 formúû
x – sin x
x ®0 x 2 x ®0 4 x 2/ 4 y Example 23 Evaluate lim .
2
x ®¥ x + cos 2 x
1 æ sin x / 2 ö 1 2 1
= lim ç ÷ = ( 1) = æ sin x ö
x ® 0 2 è x /2 ø 2 2 x ç1 – ÷
x – sin x è x ø
Sol. We have, lim = lim
1 - cos (1 - cos x ) x ®¥ x + cos 2 x x ®¥ æ cos 2 x ö
y Example 20 Solve lim x ç1 + ÷
x ®0 sin 4 x è x ø
æ xö sin x
1 - cos ç2sin 2 ÷ 1–
1 - cos (1 - cos x ) è 2ø sin 4 x = lim x =
1–0
=1
Sol. lim = lim
x ®0 sin 4 x x ®0 x4 x4 x ®¥ 2
cos x 1+0
1+
x
æ xö
1 - cos ç2sin 2 ÷ 4
è 2ø æ sin x ö
= lim lim ç ÷ sin 2 x – sin 2 y
x ®0 x4 x ®0è x ø y Example 24 Evaluate lim .
x ®y x2 – y 2
æ xö
2sin 2 çsin 2 ÷ sin 2 x – sin 2 y sin ( x + y ) sin ( x –y )
è 2ø Sol. lim = lim
= lim 1
x ®0 x4 x ®y 2
x –y 2 x ®y ( x + y ) ( x –y )
2 sin( x + y ) sin( x –y ) sin(2y )
æ æ 2 xö xö = lim ´ lim = ´1
ç sin çsin ÷ sin 2 ÷ x ®y (x +y ) x ® y ( x –y ) 2y
è 2 ø
= lim 2 × ç × 2÷
x ®0 ç 2 x x 2
÷ [as x ® y Þ ( x – y ) ® 0, but x + y ® 2y ]
ç sin 4× ÷
è 2 4 ø =
sin 2y
1 1 2y
=2´ =
42 8
Chap 05 Limits 255

y Example 25 \ a is minimum, when x = –1, i.e. a = 2


lim [( x + 5) tan -1 ( x + 5) - ( x + 1) tan -1 ( x + 1)] is equal to Again, b = lim
2 sin 2 q/ 2
= lim
2 sin 2 q/ 2
× 2
x ®¥ q®0 q2 q®04 q /4
(a) p (b) 2p
2 sin 2 q/ 2 1
p = lim × =
(c) (d) None of these q ® 0 4 (q/ 2)2 2
2
n n n –r
lim (( x + 5) tan -1( x + 5) - ( x + 1) tan -1( x + 1)) æ1ö
Sol. Here,
x ®¥
Hence, åar bn – r = å 2r × çè 2 ÷ø
r =0 r =0
ìp ü
= lim ( x + 5) í - cot -1( x + 5)ý - ( x + 1) 1
x ®¥ î2 þ Þ n
{20 + 22 + 24 + ...+ 22n }
2
ìp -1 ü
í - cot ( x + 1)ý 1 ì 1 ( 4n + 1 – 1) ü 4n + 1 – 1
î 2 þ Þ í ý =
2n î 4 –1 þ 2n ´ 3
æ 1 ö -1 æ 1 ö
tan -1 ç ÷ tan ç ÷ [i.e. sum of (n + 1) terms of GP]
è x + 5ø è x + 1ø
= lim 2p - + = 2p
x ®¥ æ 1 ö æ 1 ö
n
4n + 1 – 1
ç
è x + 5ø
÷ ç
è x + 1ø
÷ \ åar bn – r =
2n ´ 3
r =0

Hence, (b) is the correct answer.


y Example 28 Evaluate
sin ( p cos 2 x ) æxö æxö æxö æxö
y Example 26 Evaluate lim . lim cos ç ÷ cos ç ÷ cos ç ÷ ... cos ç n ÷ .
x ®0 x2 n ®¥ è2ø è4ø è8ø è2 ø
[IIT JEE 2001]
æxö æ x ö æx ö æx ö æx ö
sin ( p cos 2 x ) sin { p (1–sin 2 x )} Sol. Here, lim cos ç n ÷ cos ç n -1 ÷ ...cos ç ÷ cos ç ÷ cos ç ÷
Sol. lim = lim n ®¥ è2 ø è2 ø è8ø è4ø è2ø
x ®0 x2 x ®0 x2
sin ( p – p sin 2 x ) sin 2n A
= lim We know, cos A cos 2A cos 22 A ...cos 2n -1 A =
x ®0 x2 2n sin A
ì sin ( p sin 2 x ) p sin 2 x ü æxö æ x ö æx ö æx ö æx ö
sin ( p sin 2 x ) Thus, lim cos ç n ÷ cos ç n -1 ÷ ...cos ç ÷ cos ç ÷ cos ç ÷
= lim = lim í ´ ´ ý n ®¥ è2 ø è2 ø è8ø è4ø è2ø
x2 î p sin x
2
x ®0 x ®0 1 x2 þ
æxö
sin ( p sin 2 x ) sin 2 x sin 2n ç n ÷
= lim ´ p ´ lim è2 ø sin x
= lim = lim
x ®0 p sin 2 x x ®0 x2 n ®¥ n æxö n ®¥ æxö
2 sin ç n ÷ 2n sin ç n ÷
=1 ´ p ´ 1 = p è2 ø è2 ø
1 x
y Example 27 Let a = min { x 2 + 2x + 3, x ÎR} and = sin x × lim ×
n ®¥ n æxö x
1 – cos q n 2 sin ç n ÷
b = lim . The value of S a r ×b n – r is è2 ø
q®0 q 2 r = 0
sin x 1
= × lim ×x
Sol. Here, a = min { x 2 + 2x + 3, x Î R } x n ®¥ æxö
2n sin ç n ÷
i.e. x 2 + 2x + 3 = x 2 + 2x + 1 + 2 è2 ø

= ( x + 1) 2 + 2 sin x sin x é x q ù
= ×1 = n ® ¥, n ® 0 and lim = 1ú
x x êë 2 q ® 0 sin q û
Exercise for Session 2
1. If lim ( x -3 sin 3x + ax -2 + b ) exists and is equal to zero, then
x ®0
9 9
(a) a = - 3, b = (b) a = 3, b =
2 2
-9
(c) a = - 3, b = (d) None of these
2
x sin a - a sin x
2. The value of lim is
x ®a x -a
(a) a sin a - cos a (b) sin a - a cos a
(c) cos a + a sin a (d) sin a + a cos a
2 + cos x - 1
3. The value of lim is
x ®p ( p - x )2
1 1
(a) (b)
4 2
(c) 2 (d) Doesn’t exist
( 2 - cos q - sin q)
4. The value of lim is
q ® p/ 4 (4q - p )2
1 1
(a) (b)
16 2 16
1 1
(c) (d)
8 2 2 2
(cos x + sin x )3 - 2 2
5. The value of lim is
x ® p/ 4 1 - sin 2x
3 3
(a) (b) -
2 2
1 1
(c) (d) -
2 2
Session 3
Logarithmic Limits, Exponential Limits

Logarithmic Limits y Example 32 Evaluate lim


log e (1 + 2h ) – 2 log e (1 + h )
.
In this section, we will deal with the problems based on h ®0 h2
expansion of logarithmic series, which is given below [IIT JEE 1999]
x2 x 3 loge (1 + 2h ) – 2 loge (1 + h )
log (1 + x ) = x – + + ... ¥ Sol. We have, lim
h ®0 h2
2 3
é (2h )2
(2h )3 ù æ h2 h3 ö
where, –1 £ x £ 1 and it should be noted that the expansion ê(2h ) – + – ... ¥ ú –2 çh – + – ...÷
is true only if the base is e. To evaluate the logarithmic = lim ë 2 3 û è 2 3 ø
log (1 + x ) h ®0 h2
limit, we use lim = 1.
x ®0 x –h + 2h – ...
2 3
= lim
h ®0 h2
log { 1 + ( x – a )}
y Example 29 Evaluate lim × h 2 {–1 + 2h – ...}
x ®a (x – a) = lim
h ®0 h2
log {1 + ( x - a )} = lim {–1 + 2h – ...} = - 1
Sol. We have, lim h ®0
x ®a ( x - a)
ì 1 öü
Let x – a = y , when x ® a; y ® 0 y Example 33 Solve lim í x - x 2 × log æç 1 + ÷ ý.
x ®¥ î è x øþ
log {1 + y }
\ The given limit = lim =1 1 ì æ 1 öü
y ®0 y Sol. Here, put x = in lim í x - x 2 × log ç1 + ÷ý
y x ®¥ î è x øþ
log 10 (1 + h )
y Example 30 Evaluate lim × ì 1 log (1 + y )ü
= lim í -
h ®0 h ý
y ®0 y
î y2 þ
log10 (1 + h ) loge (1 + h ) ´ log10 e
Sol. We have, lim = lim y - log(1 + y )
h ®0 h h ®0 h = lim
loge (1 + h )
y ®0 y2
= lim ´ log10 e
h ®0 h ì y2 y3 y4 ü
y - íy - + - + Ký
é log(1 + x ) ù î 2 3 4 þ
= log10 e ´ 1 = log10 e êëQ xlim = 1ú = lim
®0 x û y ®0 y 2

log ( 5 + x ) – log ( 5 – x ) ì1 y ü
y Example 31 Evaluate lim . y 2 í - + Ký
x ®0 x = lim î 2 3 þ
ì æ x öü ì æ x öü y ®0 y2
log í5 ç1 + ÷ý – log í5 ç1 – ÷ý
î è ø
5 þ î è 5 øþ ì1 y ü 1
Sol. We have, lim = lim í - + Ký =
x ®0 x y ® 0î2 3 þ 2
æ xö æ xö
log 5 + log ç1 + ÷ – log 5– log ç1 – ÷
è 5ø è 5ø Remark
= lim
x ®0 x If we solve above question as
æ xö æ xö ì 1 log ( 1 + y ) ü ì 1 log ( 1 + y ) 1 ü
log ç1 + ÷ log ç1 – ÷ lim í - ý = ylim í - × ý
è 5ø è 5ø 1 1 2 y®0
îy y2 þ ® 0î y y yþ
= lim – = + =
x ®0 æx ö æ xö 5 5 5 é log ( 1 + y ) ù
5ç ÷ –5 ç – ÷ êas ylim = 1ú
è5ø è 5ø ë ® 0 y û
æ1 1ö
= lim ç - ÷ = 0, then it is not correct.
é log (1 + x ) ù y®0 èy yø
êëQ xlim = 1ú
®0 x û
258 Textbook of Differential Calculus

Since, limit is finite, so (a – b ) = 0 Þ b = a


Exponential Limits ax 2
There are two types of exponential limits discussed below xa + + ... ¥
2!
\ lim =2
x ®0 x
(i) Based on Series Expansion ax
Þ lim a + + ... ¥ = 2 Þ a = 2
x2 x 3 x ®0 2!
ex =1+ x + + + ... ¥
2! 3! \ b=2
To evaluate the exponential limit, we use the following a sin x - bx + cx 2 + x 3
results. y Example 38 Solve lim , if it
log (1 + x ) - 2x 3 + x 4
x ®0 2x 2
ex –1 ax –1
(a) lim =1 (b) lim = log e a exists and is finite, also find a, b and c.
x ®0 x x ®0 x a sin x - bx + cx 2 + x 3
Sol. Let L = lim
ax – b x x ® 0 2x 2 log (1 + x ) - 2x 3 + x 4
y Example 34 Evaluate lim ×
x ®0 x æ x3 x5 ö
aç x - + - ...÷ - bx + cx 2 + x 3
ax – bx ( a x –1 ) – ( b x –1 ) è 3 ! 5! ø
Sol. We have, lim = lim = lim
x ®0 x x ®0 x x ®0 æ x2 x3 x4 ö
x x 2x 2 ç x - + - + ...÷ - 2x 3 + x 4
a –1 b –1 è 2 3 4 ø
= lim – lim = log a – log b = log (a / b )
x ®0 x x ®0 x
æ aö a
(a - b )x + cx 2 + ç1 - ÷ x 3 + x 5 + ...
(ab ) x – a x – b x + 1 è 3! ø 5!
y Example 35 Evaluate lim × = lim
x ®0 2 5 1 6
x ®0 x2 x - x +...
3 2
(ab )x – a x –b x + 1 a x b x – a x –b x + 1
Sol. We have, lim = lim a
x ®0 x 2 x ® 0 x2 For finite limit, a - b = 0, c = 0, 1 - = 0, i.e. a = b = 6, c = 0]
x x
a (b – 1)–(b –1) x 3!
= lim a
x ®0 x2 + higher powers of x
x
( a – 1) ( b x –1 ) \ L = lim ! 5
= lim ´ lim = log a ´ log b x ®0 2 1
- x +L
x ®0 x x ®0 x 3 2
é ax - 1 ù a 3 a 6 3
ê x ®0
Q lim = loge a ú = × = = = [Qa = 6]
ë x û 5! 2 80 80 40
a sin x - bx + cx 2 + x 3 3
e tan x – e x So, lim =
y Example 36 Evaluate lim × x ®0 2x log (1 + x ) - 2x + x
2 3 4
40
x ® 0 tan x – x
Where, a = 6 = b, c = 0
e tan x – e x e x ´ e (tan x – x ) – e x
Sol. We have, lim = lim y Example 39 Find the values of a, b and c such that
x ® 0 tan x – x x ®0 ( tan x – x )
e x {e tan x – x – 1} axe x – b log (1 + x ) + cxe – x
= lim
x ®0 (tan x – x ) lim = 2.
x ® 0 x 2 sin x
= e 0 ´ 1 [as x ® 0, tan x – x ® 0]
axe x - b log (1 + x ) + c xe - x
=1 ´ 1 =1 Sol. We have, lim =2
x ®0 x 2 sin x
ae x – b Using the expansion, we have
y Example 37 Evaluate lim = 2 . Find a and b.
x ®0 x é æ x2 ö æ x2 x3 ö ù
êax ç1 + x + 2 ! + ...÷ –b ç x – 2 + 3 –...÷ ú
ê è
æ x x2 ö ø è ø
ú
a ç1 + + + ... ¥ ÷ –b
è ø ê æ x x2 x3 öú
ae x - b 1! 2!
ê + cx ç1 – + - + ...÷ ú
Sol. Given, lim = lim =2
x ®0 x x ®0 x êë è 1! 2! 3! øú
lim û =2
ax 2 x ®0 æ 3 5 ö
(a –b ) + xa + + ... ¥ x 2 çx –
x
+
x
–...÷
2! è ø
Þ lim =2 3! 5!
x ®0 x
Chap 05 Limits 259

æ ö px
b æa b c ö tan
y Example 42 Evaluate lim æç 2 – ö÷
x (a – b + c )– x 2 çça + – c ÷÷ + x 3 ç – + ÷ + ... a 2a
è 2 ø è2 3 2ø .
x®a è xø
lim =2
x ®0 æ x x 3 ö 5
px
x 2 çx – + –...÷ tan
è 3! 5! ø æ aö 2a
Sol. We have, lim ç2 – ÷
x ®a è xø
Now, above limit would exist, if least power in numerator is
px æ aö px
greater than or equal to least power in denominator, i.e. tan lim ç 1 – ÷ × tan
ì æ a öü 2a è xø
Coefficient of x and x 2 must be zero and coefficient of x 3 = lim í1 + ç1 – ÷ý =e x®a 2a
[1¥ form]
x ®a î è x øþ
should be 2.
æ x –a ö px
b a b c lim ç ÷ × tan lim f ( x )
i.e. a – b + c = 0, a + – c = 0, – + =2 =e x ® a
è x ø 2a
=e x ® a …(i)
2 2 3 2
On solving, we get a = 3, b = 12, c = 9. Let x –a = h , we get
æ h ö p h æ p ph ö
lim ç ÷ × tan (a + h ) lim × tan ç + ÷
èa +h ø 2a a +h è 2 2a ø
(ii) Evaluation of Exponential Limits eh ® 0 = eh ® 0
of the Form 1¥ lim
h®0
h é
a + h êë
æ ph ö ù
- cot ç ÷
è 2a ø úû
lim
h®0
–h p
× ×
1
(a + h ) tan ( ph/ 2a ) 2a p/ 2a
=e =e
To evaluate the exponential limits of the form 1 ¥ , we use –2a
×
ph/ 2a –2a
lim
h ® 0 p (a + h ) tan ( ph/ 2a ) é x ù
the following results. =e = e p (a ) = e – 2 /p ê as lim = 1ú
If lim f ( x ) = lim g( x ) = 0, ë x ® 0 tan x û
x ®a x ®a
x +4
lim
f (x)
æ x + 6ö
then lim {1 + f ( x )} 1 /g ( x )
=e x ®a g(x) y Example 43 Evaluate lim ç ÷ .
x ®a x ®¥ è x + 1ø
x +4
or when lim f ( x ) = 1 and lim g ( x ) = ¥ æ x + 6ö
x ®a x ®a Sol. We have, lim ç ÷
x ®¥ è x + 1 ø
Then, lim { f ( x )} g ( x ) = lim [1 + f ( x ) – 1]g ( x ) x +6
x ®a x ®a As x ® ¥, lim = 1 and ( x + 4 ) ® ¥ [1¥ form]
x ®¥ x + 1
lim (f ( x ) – 1 ) g ( x )
= e x ®a x +4 x +4
æ x + 6ö é æ x + 6 öù
Particular Cases \ lim ç ÷ = lim ê1 + ç –1 ÷ ú
x ® ¥ è x + 1ø x ®¥
ë è x + 1 øû
x
æ 1ö x +4 æ 5 ö
(i) lim (1 + x ) 1 /x = e (ii) lim ç 1 + ÷ = e é 5 ù lim ç
x®¥
÷ × (x + 4 )
è x + 1ø
x ®0 x ®¥ è xø = lim ê1 + =e
x ®¥ ë x + 1 úû
x x +4
æ lö lim 5 ×
é x+4 ù
(iii) lim (1 + lx ) 1 /x = e l (iv) lim ç 1 + ÷ = e l =e
x®¥ x +1
=e 5 × (1)
=e 5 ê as x ® ¥; lim = 1ú
x ®0 x ®¥ è xø x ®¥ x + 1
ë û
x
y Example 40 Evaluate lim æç 1 + ö÷ .
1
2
x®¥ è
y Example 44 Evaluate lim (1 + tan 2 2
x) x .
xø x®0
1 1
x 2 lim tan 2 x ×
lim × x
æ 2ö Sol. We have, lim (1 + tan 2
x 2
) x =e x®0 2x
Sol. We have, lim ç1 + ÷ = e x ® ¥ x = e 2 [1¥ form] x ®0
x ®¥ è xø 2
1 æ tan x ö
lim ç ÷
2 x®0 è x ø é tan x ù
y Example 41 Evaluate lim (log 3 3x )log x 3 . =e = e 1/ 2 êë as xlim = 1ú
x ®1
®0 x û
1
log x 3
= lim (log 3 3 + log 3 x )log x 3 ì p üx
y Example 45 Evaluate lim í tan æç + x ö÷ ý .
Sol. We have, lim (log 3 3x )
x ®1 x ®1

é x®0î è4 øþ
1/ log 3 x 1 ù
= lim (1 + log 3 x ) ê as logb a = log b ú [IIT JEE 1993]
x ®1 ë a û 1
1 ì p üx 1
ï tan 4 + tan x ï
lim log 3 x ´
=e
x ®1 log 3 x ì æp öü x
Sol. We have, lim í tan ç + x ÷ý = lim í ý
x ® 0î è4 øþ x ®0 p
= e1 ï 1 – tan tan x ï
î 4 þ
260 Textbook of Differential Calculus

1 1
ì 1 + tan x ü x ì 2 tan x ü x y Example 49 The value of
= lim í ý = lim í1 + ý
x ® 0 î 1 – tan x þ x ®0î 1 - tan x þ lim (2x 2 - 9 x + 8 ) cot ( 2x - 7 ) is equal to
x ® 7/2
æ 2 tan x ö 1 tan x 1
÷× lim 2× ×
(b) e -5 / 2
lim ç
x ® 0 è 1 – tan x ø x x®0 (a) e 5 / 2
=e =e x 1 – tan x
= e2
(c) e 7 / 2 (d) e 3 / 2
1 /x 2 - 7)
æ 1 + 5x 2 ö Sol. Here, lim (2x 2 - 9 x + 8)cot (2 x [1¥ form]
y Example 46 Evaluate lim ç ÷ . x ® 7/2
x ® 0 è 1 + 3x 2 ø - 7)
[IIT JEE 1996] = lim {1 + (2x 2 - 9 x + 7 )} cot (2 x
1/x 2 1/x 2 x ® 7/2
æ 1 + 5x 2 ö æ 2x 2 ö
Sol. We have, lim ç ÷ = lim ç1 + ÷ lim ( 2 x 2 - 9 x + 7 ) × cot ( 2 x - 7 )
x ® 0 è 1 + 3x 2 ø x ®0 è 1 + 3x 2 ø = e x ® 7/2
[1¥ form] 4x - 9
æ 2x 2 ö 1 lim
2 x ® 7/2
lim ç ÷. sec 2 ( 2 x - 7 ) . 2
x ® 0 è 1 + 3x 2 ø x 2 1+ 0 =e = e 5/2
=e =e = e2
x Hence, (a) is the correct answer.
æ x – 3ö
y Example 47 Evaluate lim ç ÷ . [IIT JEE 1
x ® ¥ è x + 2ø p
æ ö log x
y Example 50 The value of lim ç tan æç + log x ö÷ ÷
2000]
x x
x ®1 è è4 øø
æ x –3ö æ (– 5) ö
Sol. We have, lim ç ÷ = lim ç1 + ÷ is equal to
x ® ¥ è x + 2ø x ®¥ è x + 2ø
–5
(a) e (b) e -1
(d) e -2
æ –5 ö lim
lim ç ÷× x
x®¥
1+
2 (c) e 2
x®¥ è x + 2ø
=e = e x = e –5 1
æ æp ö ö log x
Sol. Here, lim ç tan ç + log x ÷ ÷ [1¥ form]
y Example 48 The value of x ®1 è è4 øø
p
sec 2 1
ì æ p öü 2 - bx
æ 2 tan (log x ) ö log x lim ×
2 tan (log x ) 1
lim ísin 2 ç ÷ý is equal to = lim ç1 + ÷ =e
x ® 1 {1 - tan (log x )} log x

x ®0
î è 2 - ax ø þ x ®1 è 1 - tan (log x ) ø

(a) e - a /b (b) e - a
2
/b 2 tan (log x ) 1
(c) a 2 a /b (d) e 4 a /b 2 lim ×
1 - tan (log x )
= e 2 × (1) = e 2
x ®1
æ p ö =e log x
sec 2 ç ÷
æ æ p öö è 2 - bx ø [as x ® 1, log x ® 0]
Sol. Here, lim çsin 2 ç ÷÷
x ®0 è è 2 - ax ø ø Hence, (c) is the correct answer.
æ p ö 2m
sec 2 ç ÷
y Example 51 The value of lim æç sin + cos ö÷
ì æ p öü è 2 - bx ø x 3x x
= lim í1 - cos 2 ç ÷ý [1¥ form ]
x ®0 è 2 - ax øþ x ®0 è m mø
î
ì ü is equal to
ï ï
ï 2 æ
lim - í cos ç
p ö
÷×
1 ï
ý (a) e (b) 1 (c) e -1 (d) e 2
x®0
ï è 2 - ax ø æ p öï 2m
cos 2 ç ÷
ïî è 2 - bx ø ïþ
=e ï æ x 3x ö x
Sol. Here, lim çsin + cos ÷
ì æ p ö
ç
æ p ö
÷ cos ç ÷
pa
´
ü x ®0 è m mø
ï 2 sin ï
ï è 2 - ax ø è 2 - ax ø ( 2 - ax ) 2 ï 2m
lim - í × ý
pb
x®0
ï 2 sin æ p ö æ p ö
÷ ´ ï ì æ x 3x öü x
ïî
ç ÷ cos ç
è 2 - bx ø è 2 - bx ø ( 2 - bx ) 2 ïþ = lim í1 + çsin + cos - 1÷ý [1¥ form ]
=e ï x ® 0î è m m øþ
[using L’Hospital’s rule] 2m æ x
+ cos
3x ö
- 1÷
lim ç sin
x è ø
æ 2p ö = ex® m m
0
sin ç ÷
è 2 - ax ø a ( 2 - bx ) 2
- lim × × æ1 æxö 3 3x ö
x®0 æ 2 p ö b ( 2 - ax ) 2 a ( 2 - bx ) 3 2m ç cos ç ÷ - sin ÷
sin ç ÷ - lim × a èm èm ø m m ø æ1ö
è 2 - bx ø x®0 b ( 2 - ax ) 3 - lim 2m ç ÷
=e =e =e b
= ex®0 1 = e
èm ø
= e2
Hence, (a) is the correct answer. Hence, (d) is the correct answer.
Chap 05 Limits 261

n æ n b - 1ö
æa - 1+ n b ö lim ç ÷ ×n
[1¥ form ]
y Example 52 The value of lim ç ÷ =e
n ® ¥è a ø
æ - 1ö
n ®¥ è a ø 1/n
b log b ç 2 ÷
1 b1/n - 1 1 èn ø
lim × lim ×
(a > 0, b > 0) is equal to =e n®¥a 1/n
=e
n®¥ a -1/n 2
a b
(a) b (b) a (c) b (d) a 1
log e b 1/ a

æa - 1 + b ö n æ
n
b - 1ö n
n = ea = e log e b = b1/a
Sol. Here, lim ç ÷ = lim ç1 + ÷ Hence, (a) is the correct answer.
n ®¥ è a ø n ® ¥ è a ø

Exercise for Session 3


e x - e x cos x
1. The value of lim is
x ® 0 x + sin x

(a) 0 (b) 1 (c) -1 (d) None of these


x -y
y x
2. The value of lim is
x ®y xx - yy
1 - log x 1 - log y log x - log y
(a) (b) (c) (d) None of these
1 + log x 1 + log y log x + log y

px - q x
3. The value of lim is
x ®0 r x - sx
1 - log p log p - log q log p × log q
(a) (b) (c) (d) None of these
1 + log p log r - log s log r × log s

4. The value of lim ( x + 2) tan-1 ( x + 2) - ( x tan-1 x ) is


x ®¥
p p
(a) (b) Doesn’t exist (c) (d) None of these
2 4
(cos a )x - (sin a )x - cos 2a æ pö
5. The value of lim , a Î ç0, ÷ is
x ®4 x -4 è 2ø
(a) log (cos a) + (sin a)4 log (sin a) (b) (cos4 a) log (cos a) - (sin a)4 log (sin a)
(c) (cos4 a) log (cos a) (d) None of these
a/ x
æ 1x + 2x + 3x + . . . + n x ö
6. The value of lim ç ÷ is
x ®0è n ø
(a) (n !)a / n (b) n ! (c) a n! (d) Doesn’t exist
7. If lim (1 + ax + bx ) 2 2/ x
= e , the values of a and b are
3
x ®0
3 1
(a) a = , b ÎR (b) a = , b ÎR (c) a ÎR, b ÎR (d) None of these
2 2
-a
8. If a and b are roots of ax 2 + bx + c = 0, the value of lim (1 + ax 2 + bx + c )2/ x is
x ®a
2a
( a -b )
(a) e 2 a( a - b ) (b) e a( a - b ) (c) e 3 (d) None of these
9. . ) + [(1 + 0.0001)
The value of lim ((15 n 10000 n 1/ n
] ) , where [×] denotes the greatest integer function, is
n®¥
1
(a) 1 (b) (c) Doesn’t exist (d) 2
2
10. The value of lim | x |[cos x ] , where [×] denotes the greatest integer function, is
x ®0

(a) 0 (b) Doesn’t exist (c) 1 (d) None of these


Session 4
Miscellaneous Forms
Miscellaneous Forms = lim
n ®¥
log e – 0
[by L’Hospital’s rule]
1
(i) 00 Form log A = 1
1/n
When lim f ( x ) ¹ 1 but f ( x ) is positive in the æen ö
x ®a Þ A = e or lim ç ÷
1
=e
n ® ¥è p ø
neighbourhood of x = a.
In this case, we write { f ( x )} g ( x ) = e loge {f ( x )}
g (x )
y Example 56 Evaluate lim (cosec x ) x .
x ®0
lim g ( x ) loge f ( x ) Sol. Let A = lim (cosec x )x [¥ 0 form]
Þ lim [ f ( x )]g ( x ) = e x ® a x ®0
x ®a
Þ log A = lim x log ( cosec x )
x ®0
sin x
y Example 53 Evaluate lim | x | . log ( cosec x ) é¥ ù
x ®0 = lim êë ¥ form úû
log e | x |
x ®0 1
lim
Sol. lim | x | sin x = lim e sin x log e | x | = e x ® 0 cosec x [0° form] x
x ®0 x ®0 1
× (– cosec x cot x )
1/ x cosec x
lim
x®0 - cos ec x cot x = lim
=e [by L’ Hospital’s rule] x ®0 1
2
– 2
sin x 2 æ sin x ö æ x ö x
lim - lim - ç ÷ ×ç ÷
x®0 è x ø è cos x ø [by L’Hospital’s rule]
=e x®0 x cos x =e x2
- (1) 2× ( 0 )
= lim =0
=e =e =10 x ® 0 tan x

\ log A = 0 or A = 1 Þ lim ( cosec x )x = 1


y Example 54 Evaluate lim ( pn ) 2/n . x ®0
n ®¥

Sol. Let A = lim ( pn )2 /n


n ®¥
[¥ 0 form] (ii) 10 Form
\ log A = lim
2 log ( pn ) é¥ ù (1 + x )1/x - e
n ®¥ n êë ¥ form úû y Example 57 Solve lim .
x ®0 x
1
2× ×p (1 + x )1/x - e
pn Sol. Let L = lim
= lim [by L’Hospital’s rule] x ®0 x
n ®¥ 1 ìï log (1 + x ) -1üï
2 e íe x ý
= lim =0
e log (1 + x )
1/x
n ®¥ n -e ïî ïþ
= lim = lim
\ loge A = 0 Þ A = 1 x ®0 x x ®0 x
1 /n e {e M - 1} log(1 + x ) - x
æen ö = lim × lim ,
y Example 55 Evaluate lim ç ÷ . x ®0 M x ®0 x2
n ®¥ è p ø
log(1 + x )
where M = -1
1/n x
æen ö
Sol. Let A = lim ç ÷ [¥ 0 form] 1
n ® ¥è p ø -1
e × {e M - 1} 1+ x
\ L = lim × lim
1 æen ö M ®0 M x ®0 2x
\ log A = lim log ç ÷
n ® ¥n èpø -x e
= e ´ 1 ´ lim = - [as x ® 0 Þ M ® 0]
x ® 0 2x ( 1 + x ) 2
n log e – log p é¥ ù
= lim êë ¥ form úû
n ®¥ n
Chap 05 Limits 263

Exercise for Session 4


1. Evaluate lim + (sin x )x . 2. Evaluate lim + (sin x )tanx .
x ®0 x ®0

æe n ö 1 /n
3. Evaluate lim ç ÷ .
n® ¥è p ø

e - (1 + x )1/ x
4. The value of lim is
x ®0 tan x
11e e
(a) e (b) (c) (d) None of these
24 2

Session 5
Left Hand and Right Hand Limits
Left Hand and Right Hand Limits Now, RHL of f ( x ) at x = 4
Let y = f ( x ) be a given function and x = a is the point = lim + f ( x ) = lim f ( 4 + h )
x ®4 h ®0
under consideration. Left tendency of f ( x ) at x = a is
| 4 + h – 4| |h |
called its left hand limit and right tendency is called its = lim = lim =1
h ®0 4 + h – 4 h ®0 h
right hand limit.
LHL of f ( x ) at x = 4
Left tendency (left hand limit) is denoted by f (a – 0 ) or
= lim - f ( x ) = lim f ( 4 – h )
f (a–) and right tendency (right hand limit) is denoted by x ®4 h ®0
f (a + 0 ) or f (a +) and are written as |4 – h – 4| |h | h
= lim = lim = lim = -1
h ® 0 – h h ® 0 -h
f (a – 0 ) = lim f (a – h ) ü h ®0 4 –h – 4
h®0 ï
ý Thus, RHL ¹ LHL. So, lim f ( x ) does not exist.
f (a + 0 ) = lim f (a + h )ï x ®4
h®0 þ
where, h is a small positive number. ì 5x – 4, 0 < x £ 1
y Example 59 If f ( x ) = í 3 , show that
Thus, for the existence of the limit of f ( x ) at x = a, it is î4 x – 3x , 1 < x < 2
necessary and sufficient that lim f ( x ) exists.
f (a – 0 ) = f (a + 0 ) , if these are finite or f (a – 0 ) and x ®1
ì5x – 4, 0 < x £ 1
f (a + 0 ) both should be either + ¥ or –¥. Sol. We have, f ( x ) = í 3
î 4 x – 3x , 1 < x < 2
y Example 58 Evaluate the right hand limit and left LHL of f ( x ) at x = 1 = lim - f ( x ) = lim f (1 – h )
x ®1 h ®0
hand limit of the function
= lim 5(1 – h )– 4 = lim 1 – 5h = 1
ì| x – 4 | h ®0 h ®0
ï ,x ¹4
f (x ) = í x – 4 . RHL of f ( x ) at x = 1 = lim + f ( x ) = lim f (1 + h )
ï0, x ®1 h ®0
î x = 4
= lim 4 (1 + h ) –3(1 + h ) = 4 (1)3 – 3 (1) = 1
3
ì| x – 4 | h ®0
ï ,x ¹4
Sol. Given, f ( x ) = í x – 4 Thus, RHL = LHL = 1. So, lim f ( x ) exists and is equal to 1.
ï0, x =4 x ®1
î
264 Textbook of Differential Calculus

e 1/x – 1
y Example 60 Show that lim does not exist. é sin x ù
x ®0 e 1/x + 1 y Example 62 Solve (i) lim [sin –1 x ] (ii) lim + ê
e 1/x
–1 x ®1 x ®0 ë x ú û
Sol. Let f (x ) = .
e 1/x
+1 é sin x ù
(iii) lim - ê
Then, LHL = lim - f ( x ) = lim f (0 – h ) x ®0 ë x ú û
x ®0 h ®0
e –1/h – 1 (where [ × ] denotes greatest integer function.)
= lim
h ®0 e –1/h
+1 Sol. (i) Here, L = lim [sin –1 x ]
x ®1
(1/e 1/h – 1) 0–1
= lim = = –1 Put sin x = t –1
h ®0 (1/e 1/h + 1) 0 + 1
p
1 \ x = sin t and t ® as x ® 1
[as h ® 0 Þ ® ¥ Þ e 1/h ® ¥ Þ 1/e 1/h ® 0 ] ...(i) 2
h
épù
RHL = lim + f ( x ) = lim f (0 + h ) Þ L = lim [t ] = ê ú = 1
x ®0 h ®0 t ® p/ 2 ë2û

e 1/h – 1 (1–1/e 1/h ) \ lim [sin –1 x ] = 1


= lim = lim x ®1
h ®0 e 1/h + 1 h ®0 (1 + 1/e 1/h )
é sin x ù
[dividing numerator and denominator both by e 1/h ] (ii) Here, L = lim + ê
x ®0 ë x ú û
1–0
= =1 [using Eq. (i)] sin x
1+0 Put = t Þ t ® 1– as x ® 0+ and [ t ] = 0
x
Clearly, lim - f ( x ) ¹ lim + f ( x )
x ®0 x ®0
Þ L = lim- [t ] = [1 – h ] = 0
t ®1
Hence, lim f ( x ) doesn’t exist. [as x ® 0 + h Þ t ® 1 – h ]
x ®0

1 – cos 2 ( x – 1) é sin x ù
(iii) Here, L = lim - ê
y Example 61 Evaluate lim . x ®0 ë x ú û
x ®1 ( x – 1)
sin x
[IIT JEE 1998] Put =t
x
1 – cos 2 ( x – 1)
Sol. We have, lim Þ t ® 1– as x ® 0–
x ®1 ( x – 1)
\ L = lim- [t ] = [1 – h ] = 0
2sin 2 ( x – 1) 2 |sin ( x – 1)| t ®1
= lim = lim [as x ® 0 – h Þ t ® 1 – h \ [t ] = 0]
x ®1 ( x – 1) x ®1 ( x – 1)
2 |sin ( x – 1)| y Example 63 Solve (i) lim [tan -1 x ]
\ LHL = lim - x ®¥
(ii) lim [tan -1 x ]
x ®1 ( x – 1)
x ®-¥
2 |sin (– h )| 2 sin h
= lim = lim =– 2 (where [ × ] denotes greatest integer function.)
h ®0 (– h ) h ®0 –h
Sol. (i) Here, lim [tan –1 x ]
2 |sin ( x – 1)| x ®¥
Again, RHL = lim +
x ®1 ( x – 1) p
Put tan x = t Þ t ®
–1
as x ® ¥
2
2 |sin (h )|
= lim épù
h ®0 h \ lim [ t ] = ê ú = 1
t ® p/ 2 ë2û
2 sin h (ii) Here, lim [tan –1 x ]
= lim = 2 x ®- ¥
h ®0 h
p
Clearly, lim f ( x ) ¹ lim - f ( x ) Put tan –1 x = t Þ t ® – as x ® – ¥
x ® 1+ x ®1 2
é pù
Hence, lim f ( x ) doesn’t exist. \ lim [t ] = ê– ú = [–1.57] = – 2
x ®1
t ® - p/ 2 ë 2û
Chap 05 Limits 265

y Example 64 Solve (i) lim + é


tan x ù Sol. (i) Here, lim [cot x ]
x ®0
x ®0 ë x ú
ê û Put cot x = t , now as x ® 0 ; cot x exhibits two values
é tan x ù for x ® 0+ and x ® 0–. i.e. cot x ® + ¥ and
(ii) lim - ê
x ®0 ë x ú û cot x ® – ¥, respectively.
(where [ × ] denotes greatest integer function.) \ We should apply right hand and left hand limit;
é tan x ù i.e. lim + [cot x ] = lim [t ] = ¥
Sol. (i) Here, lim + ê úû x ®0 t ®+ ¥
x ®0 ë x
[Q cot x = t Þ t ® + ¥ as x ® 0+ ]
tan x
Put = t Þ t ® 1+ as x ® 0+ and lim - [cot x ] = lim [t ] = – ¥
x x ®0 t ®- ¥

\ lim+ [ t ] = [1 + h ] = 1 [as x ® 0 + h Þ t ® 1 + h ] [Q cot x = t Þ t ® – ¥ as x ® 0–]


t ®1
é tan x ù \ Limit doesn’t exist.
(ii) Here, lim - ê úû
x ®0 ë x (ii) Here, lim [cot –1 x ] = lim+ [t ]
x ®+ ¥ t ®0
tan x
Put = t Þ t ® 1+ as x ® 0–
x [Q cot –1
x = t Þ t ® 0+ as x ® + ¥ ]
\ lim+ [ t ] = [1 + h ] = 1 [as x ® 0 – h Þ t ® 1 + h ] = lim [0 + h ] = lim 0 = 0
t ®1 h ®0 h ®0

y Example 65 Solve é | x |ù
-1 -1 y Example 67 Solve lim êsin ú , where [ × ] denotes
(i) lim- [sin (sin x )] (ii) lim [sin (sin x )] x ®0ë x û
x ®1 x ® p/ 2

(where [× ] denotes greatest integer function.) greatest integer function.


é | x |ù
Sol. (i) Here, lim - [sin (sin –1 x )] Sol. Here, lim êsin , since we have greatest integer
x ®1 x ®0ë x úû
= lim - [ x ] [as sin (sin –1 x ) = x , if –1 £ x £ 1] function, we must define function.
x ®1 é sin | 0 + h | ù
Now, RHL (put x = 0 + h ) = lim ê
= lim [1 – h ] [as x ® 1– Þ x = 1 – h ] h ®0 ë 0 + h úû
h ®0
sin h
We know that, ® 1 as h ® 0 but less than 1.
=0 h
\ lim [sin (sin –1 x )] = lim - [sin (sin –1 x )] é æ sin h ö ù
x ®1 x ®1 \ RHL = lim 0 = 0 êQ çè h ÷ø = 0 as h ® 0ú
h ®0 ë û
and no need to check for lim + [sin (sin –1 x )]
x ®1 é | 0 –h | ù
Again, LHL (put x = 0 – h ) = lim êsin ,
(ii) lim [sin (sin x )] = lim - [sin (sin x )]
–1 –1 h ®0 ë 0–h úû
p sin h
x ® x ®
p we know ® – 1 as h ® 0 but greater than –1.
2 2 –h
ép ù é æ sin h ö ù
= lim - [ x ] = lim ê – h ú = 1 \ LHL = lim – 1 = –1 êQ çè h ÷ø = - 1 as h ® 0 ú
p h ®0 ë 2 û h ®0
x ® ë û
2
Thus, Limit doesn’t exist, as RHL = 0 and LHL = –1.
Remark
é sin | x | ù
If lim [sin (sin–1 x )], it means you have to calculate only left
x ®1
y Example 68 Solve lim ê
x ®0 ë |x | û
ú , where [ × ] denotes
hand limit and not right hand limit as for x > 1, sin (sin–1 x ) is
not defined.
greatest integer function.
é sin | x | ù
Sol. Here, lim ...(i)
y Example 66 Solve (i) lim [cot x ] x ®0ê
ë | x | úû
x ®0
sin x sin | x |
(ii) lim [cot–1 x ] We know that, ® 1 as x ® 0 or ® 1 as x ® 0
x ®+¥ x |x |
from right or left,
(where [ × ] denotes greatest integer function.) sin| x |
i.e. at x = 0 + h or x = 0 – h , ® 1–
|x |
266 Textbook of Differential Calculus

é sin | x | ù sin | x | (a) -1 (b) 4


\ ê | x | ú =0 as | x | < 1
ë û (c) 5 (d) None of these
From Eq. (i) whether we find RHL or LHL Sol. We know, when x ® 0
x -x
é sin | x | ù é sin | x | ù Þ <1 Þ > -1
lim ê ú = lim 0 = 0 Þ lim ê ú, tan x tan x
x ® 0 ë |x | û x ®0 x ® 0 ë |x | û
- 2x
exists and is 0. Þ > -2
tan x
é - 2x ù é - 2x ù
= -2
y Example 69 lim ê ú , where [ × ] denotes
So, lim ê
x ® 0 ë tan x û x ® 0 ë tan x ú
û
greatest integer function is Hence, (d) is the correct answer.

Exercise for Session 5


1. The value of lim {1 - x + [ x - 1] + [1 - x ]} (where [ × ] denotes the greatest integral function) is
x ®1

(a) -1 (b) Doesn’t exist


(c) 1 (d) None of these
sin [ x ]
2. The value of lim (where [ × ] denotes the greatest integer function) is
x ®0 [x ]
(a) 1 (b) sin 1
(c) Doesn’t exist (d) None of these

3. The value of lim sin-1 {x } (where { × } denotes fractional part of x ) is


x ®0
p
(a) 0 (b)
2
(c) Doesn’t exist (d) None of these
é x2 ù
4. The value of lim ê
x ® 0 sin x tan x
ú (where [ × ] denotes the greatest integer function) is
ë û
(a) 0 (b) 1
(c) Doesn’t exist (d) None of these
Session 6
Use of Standard Theorems/Results,
Use of Newton-Leibnitz’s Formula in Evaluating
the Limits, Summation of Series Using Definite
Integral as the Limit

Use of Standard Theorem 2 :


Theorems/Results Limits of Trigonometric Functions
If x is small and is measured in radians, then
Theorem 1 : lim
sin x
= 1 = lim
x
Sandwich/Squeeze Play Theorem x ®0 x x ® 0 sin x

General The Squeeze principle is used on limit problems sin -1 x x


= lim x cosec x = lim = lim
where the usual algebraic methods, factorisation or x ®0 x ®0 x x ® 0 sin -1 x
algebraic manipulation etc., are not effective. However, it Proof Consider a circle with unit radius.
requires to “squeeze” our problem in between two other
simpler function, whose limits can be easily computed and Area of DOAP < Area of sector OAP < Area of DOAT
equal. Use of Squeeze principle requires accurate analysis,
T
indepth algebra skills and careful use of inequalities.
Statement If f , g and h are three functions such that Y
P

tan x
f ( x ) £ g ( x ) £ h ( x ) for all x in some interval containing
the point x = c and if 1
sin x
x
lim f ( x ) = lim h ( x ) = L X′
O cos x A
X
x ®c x ®c
1
Y
x2
h (x) =1+ –
2 Y′
Figure 5.11
y = g(x) (0, 1)
sin x x tan x x 1
x2 < < Þ 1< < [Q0 < x < p/ 2 ]
f(x) =1 – –
4
2 2 2 sin x cos x
sin x
XN O X Þ cos x < <1
x
YN Now, using Sandwich theorem,
Figure 5.10 sin x
lim cos x < lim+ <1
x ®0 x ®0 x
Then, lim g ( x ) = L
x ®c sin x
Obviously, we have lim+ =1
From the figure, note that lim g ( x ) = 1. x ®0 x
x ®0
sin y
Remark Put x = - y , lim =1
y ® 0- y
The quantity c may be a finite number, + ¥ or - ¥. Similarly, L may
also be finite number, + ¥ or - ¥. sin x
Hence, lim =1
x ®0 x
268 Textbook of Differential Calculus

sin x x y Example 72 Evaluate


Similarly, lim = 1 = lim = lim x cosec x
x ®0 x x ® 0 sin x x ® 0 æ n n n n ö
lim ç 2 + 2 + 2 + ... + 2 ÷.
sin -1 x x n ®¥ è n + 1 n + 2 n + 3 n + nø
= lim = lim …(i)
x ®0 x x ® 0 sin -1 x
n n n n
Sol. Let f (n ) = + + + ... +
Using Eq. (i), we can deduce n +1
2
n +2
2
n +3
2
n +n
2

tan x x
lim = 1 = lim = lim x cot x Note that f (n ) has n terms which are decreasing.
x ®0 x x ® 0 tan x x ®0
Suppose
tan -1 x x æ n n n n ö
= lim = lim h (n ) = ç 2 + 2 + 2 + ... + 2 ÷ , n terms
x ®0 x x ® 0 tan -1 x èn + 1 n + 1 n + 1 n + 1ø
n2
Important Results = [obviously f (n ) < h (n )]
n +1
2
sin x
The lim always approaches 1 from its left hand,
x®0 x and
i.e. 0.9999.... æ n n n n ö
g (n ) = ç 2 + 2 + 2 + ... + 2 ÷, n terms
Þ lim éê
sin x ù
= 0, where [ × ] denotes step up function. èn + n n + n n + n n +nø
x®0 ë x ú û
é æ sin x ö ù n2
êNote that çè xlim ÷ = 1ú = [obviously g (n ) < f (n )]
ë ® 0 x ø û n2 + n
tan x
Note that the lim
x®0 x
approaches 1 from RHS. Hence, lim g (n ) < lim f (n ) < lim h (n )
n®¥ n®¥ n®¥
Þ lim éê ù = 1, where [ × ] denotes step up function.
tan x
x®0 ë x ú û Since, lim g (n ) = 1 = lim h (n )
n ®¥ n ®¥

2 Hence, using Sandwich theorem, lim f (n ) = 1.


y Example 70 Evaluate lim x 3 cos . n ®¥
x ®0 x x éb ù
y Example 73 The value of the lim (a ¹ 0)
2 x ®0 a êë x úû
Sol. Here, lim x 3 cos
x ®0 x (where [ × ] denotes the greatest integer function) is
2 2
As, - 1 £ cos £ 1 Þ - x 3 £ x 3 cos £ x 3 for x > 0 (a) a (b) b
x x b b
2
(c) (d) 1 -
and x £ x cos £ - x for x < 0,
3 3 3 a a
x b éb ù b
2 Sol. Since, -1< ê ú £
Thus, lim x 3 cos = 0, as in both the cases limit is zero. x ëx û x
x ®0 x x
Now, we have two cases depending upon the value of .
a
x 2 (2 + sin 2 x ) x
Case I For > 0
y Example 71 Evaluate lim . a
x ®¥ x + 100
æb ö x éb ù x b x
x 2 (2 + sin 2 x ) Þ lim ç - 1÷ < lim ê ú £ lim ×
Sol. We have, lim x ®0 è x ø a x ®0 ë x û a x ® 0 x a
x ®¥ x + 100
x éb ù b
2x 2 x 2 (2 + sin 2 x ) 3x 2 Using Squeeze play theorem, we have = lim =
Now, £ £ , as 0 £ sin 2 x £ 1 x ®0 a êë x úû a
x + 100 x + 100 x + 100
x
2x 2 x 2 (2 + sin 2 x ) 3x 2 Case II For <0
\ lim £ lim £ lim a
x ® ¥ x + 100 x ®0 x + 100 x ® ¥ x + 100
æb öx éb ù x b x
Þ lim ç - 1÷ > lim ê ú ³ lim ×
x 2 (2 + sin 2 x ) x ®0 è x ø a x ®0 ë x û a x ® 0 x a
Þ lim =¥
x ®¥ x + 100 Using Squeeze play theorem, we have
é 2x 2 3x 2 ù x éb ù b
êQxlim = lim = ¥ú lim =
®¥ x + 100 x ®¥ x + 100 ë x úû a
x ®0 a ê
ë û
Hence, (c) is the correct answer.
Chap 05 Limits 269

y Example 74 Evaluate Use of Newton-Leibnitz’s Formula


[ x ] + [2x ] + [ 3x ] + ...+ [nx ]
lim
n ®¥ n2
, in Evaluating the Limits
Let us consider the definite integral,
where [ × ] denotes the greatest integer function. y (x)
I (x ) = ò f (t ) dt
f (x)
Sol. We know that, x – 1 < [x ] £ x
Newton-Leibnitz’s formula states that,
Þ 2x – 1 < [2x ] £ 2x d ìd ü ìd ü
Þ 3x – 1 < [3x ] £ 3x { I ( x )} = f { y ( x )} × í y ( x )ý – f { f ( x )} í f ( x )ý
.............................
dx îdx þ îdx þ
.............................
Þ nx – 1 < [nx ] £ nx y Example 75 Evaluate
\ ( x + 2x + 3x + ... + nx ) – n < [ x ] + [2x ] + ... + [nx ] æ 1 x 2 1 1 ö
£ ( x + 2x + ... + nx ) lim ç 5 ò e –t dt – 4 + 2 ÷.
x®0 è x 0 x 3x ø
xn (n + 1) n
x × n ( n + 1)
Þ – n < å [r x ] £ æ 1 x -t 2 1 1 ö
2 r =1 2 Sol We have, lim ç 5
x ®0 è x ò0 e dt -
x4
+ ÷
3x 2 ø
[ x ] + [2x ] + ...+ [nx ] x
Thus, lim 3 ò e –t dt – 3x + x 3
2
n ®¥ n2 0 é0 ù
= lim êë 0 form úû
xæ 1ö 1 [ x ] + [2x ] + ... + [nx ] x ®0 3x 5
Þ lim ç1 + ÷ – < lim
n ®¥ 2 è ø
n n n ® ¥ n2 d x
ò0
2

xæ 1ö 3 e –t dt – 3 + 3x 2
£ lim ç1 + ÷ = lim dx [by L’Hospital’s rule]
n ®¥ 2 è nø x ®0 15x 4
x [ x ] + [2x ] + ...+ [nx ] x Applying Newton-Leibnitz’s formula,
Þ < lim £
2 n ®¥ n2 2 d x –t 2 d d
dx ò 0
2 2
e dt = e – x × (x ) – e –0 ( 0) = e – x
[ x ] + [2x ] + ...+ [nx ] x dx dx
\ lim =
n ®¥ n2 2 d x 2
3 ò e –t dt – 3 + 3x 2 2

Aliter We know that, [ x ] = x – { x } dx 0 3e – x – 3 + 3x 2


\ lim = lim
n x ®0 15x 4 x ®0 15x 4
år x = [x ] + [2x ] + ...+ [nx ] é0 ù
r =1 êë 0 form úû
= x – { x } + 2x – { 2x } + ... + nx –{ nx }
2
= ( x + 2x + 3x + ...+ nx ) – ({ x } + { 2x } + ... + { nx }) – 3(2x )e – x + 6x
xn (n + 1) = lim [again, apply L’Hospital’s rule]
= – ({ x } + { 2x } + ... + { nx }) x ®0 60 x 3
2 2
–6 x ( e – x – 1 )
2
–(e – x – 1) æe –x 2 – 1ö
1 n x æ 1 ö { x } + { 2x } + ...+ { nx } = lim = lim =
1
ç ÷
\ 2 å [rn ] = ç1 + ÷ – lim ç –x 2 ÷
n r =1 2 è nø n2 x ®0 60x 3 x ®0 10x 2 10 x ® 0 è ø
Since, 0 £ { rx } < 1 1 1 é æe x - 1ö ù
n = ´1 = êQ lim ç ÷ = 1ú
\ 0£ å {rx } < n 10 10 êë x ®0 è x ø úû
r =1
x
x – ò cos t 2 dt
n
å {rx } y Example 76 Evaluate lim
0
.
r =1
Þ lim 2
=0 x®0 x 3 – 6x
n ®¥ n
x
x – ò cos t 2 dt
n n
å[rx ] å { rx } Sol. Let L = lim
0 é0 ù
r =1 x æ 1ö r =1 x ®0 3 êë 0 form úû
\ lim = lim ç1 + ÷ – lim x – 6x
n ®¥ n2 n ®¥ 2 è n ø n ® ¥ n2
Applying L’Hospital’s rule, we get
n
å[rx ] 1–
d x
ò 0 cos t dt
2
r =1 x L = lim dx
Þ lim =
n ®¥ n2 2 x ®0 3x 2 – 6
270 Textbook of Differential Calculus

Applying Newton-Leibnitz’s rule, 1 2n


where, a = lim = 0 and b = lim =2
d x n ®¥ n n ®¥ n

dx ò 0
(cos t 2 ) dt = cos ( x 2 ) × 1 – 0 = cos( x 2 ) 2n
1 r/n x
å
2
d x \ L = lim = ò0 dx
ò 0 cos t dt
n ®¥
1– 2 n r =1 1 + (r/n ) 2
1 + x2
dx 1 – cos ( x 2 )
\ L = lim = lim 2 2
x ®0 3x 2 – 6 x ® 0 3( x 2 – 2) = ( 1 + x )0 = 5 – 1
1 – cos 0 1 – 1 0
= = = =0 y Example 79 Evaluate
3 (0 – 2) 3 (–2) –6
æ n n n ö
x2 lim ç 2 2 + 2 + + ÷.
ò0 cos t 2 dt K
n ®¥ èn + 1 n + 22 n2 + n2 ø
y Example 77 Evaluate lim .
x®0 x sin x æ n n n ö
Sol. Let S = lim ç 2 + 2 + ... + 2 ÷
[IIT JEE 1997] n ® ¥ èn + 1 2
n +2 2
n + n2 ø
Sol. Applying Newton-Leibnitz’s rule, followed by L' Hospital’s n
n 1 n n2
rule, we get = lim
n ® ¥ r = 1n2
å +r2
= lim å
n ® ¥ n r = 1n2 + r 2
cos ( x 2 )2 × { 2x } - 0 2 cos x 4
lim = lim [dividing numerator and denominator both by n]
x ®0 x cos x + sin x x ®0 æ sin x ö
cos x + ç ÷
è x ø 1 n 1
2 cos 0 2 = lim
n ®¥ n
å 1 + (r/n )2
= = =1 r =1
cos 0 + 1 1 + 1
n
r 1
= x ; = dx ; lim å = ò
b
Replace
n n n ®¥r =1 a

Summation of Series Using where, a = lim


1
= 0 and b = lim
n
=1
Definite Integral as the Limit 1
n ®¥ n
n
1
x ®¥ n

The expression of the form, we get, S = lim


n ® ¥n
å 1 + (r/n )2
y (x) r =1
1 ær ö b
lim
n®¥ n
å f ç ÷ = ò f ( x ) dx
ènø a =ò
1 1
dx =[tan –1 x ] 10 = tan –1(1) – tan –1(0) =
p
r = f (x) 01 +x 2
4
where, (i) S is replaced by ò,
1/n
y Example 80 The value of lim æç n ö÷
n!
r is equal to
(ii) is replaced by x, n ®¥ èn ø
n
1 1 1
(iii) is replaced by dx, (a) (b) e (c) e 2 (d)
n e e2
f (x ) y (x ) 1/n
(iv) To obtain, a = lim and b = lim æ n! ö
n®¥ n n®¥ n Sol. Let A = lim ç n ÷
n ® ¥ èn ø

The value so obtained is the required sum of the given 1 æ n ( n - 1) ( n - 2) 3 2 1ö


series. \ log A = lim log ç × × ... × × ÷
n ®¥ n èn n n n n nø
1 2n r
y Example 78 Evaluate lim å . 1 n -1
æn - r ö
n ®¥ n
r =1 n + r
2 2 = lim
n ® ¥n
å log çè n ø
÷
r =0
[IIT JEE 1997] 1
1 2n
r = ò0 1 × log (1 - x ) dx = - 1
Sol. Let L = lim
n ®¥ n
å dividing numerator and
n -1
n2 + r 2
r =1 r 1 0
by n and by dx lim å =
b

denominator both by n, we get


[replace
n n n ®¥
r =0
òan
= 0,
, where

1 2n
r/n n -1
L = lim
n ®¥
å a = lim , b = lim
n ®¥ n ®¥ n
= 1]
n r =1 1 + (r/n )2
2n Þ loge A = - 1 Þ A = e -1
r 1
Put = x ; = dx ; lim å = ò
b

n n n ®¥r =1 a Hence, (a) is the correct answer.


Chap 05 Limits 271

Exercise for Session 6


1 2
1. The value of lim ([1 x + 12 ] + [22 x + 22 ] + . . . + [n 2x + n 2 ]) (where [ × ] denotes the greatest integer
n®¥ n3
function) is
x 1
(a) (b) x +
3 3
x 1
(c) + (d) None of these
3 3
ì [12 (sin x )x ] + [22 (sin x )x ] + . . . + [n 2 (sin x )x ]ü
2. The value of lim í lim ý (where [ × ] denotes the greatest integer
x ®¥ n®¥
î n3 þ
function) is
x sin x x
(a) + (b) + (sin x )x
3 3 3
1
(c) (d) 0
3
x
ò1 | t - 1| dt
3. The value of lim + is
x ®1 sin ( x - 1)
(a) 0 (b) 1
(c) Doesn’t exist (d) None of these
n 1/ n
æ Kö
4. The value of lim
n®¥
å log çè1 + n ÷ø is
K =1

(a) loge æç ö÷ (b) loge æç ö÷


e 4
è 4ø èe ø
(c) loge 4 (d) None of these
æ 1 1 1 1ö
5. The value of lim ç + + + ... + ÷ is
n ® ¥ è na na + 1 na + 2 nb ø

(a) log æç ö÷ (b) log æç ö÷


a b
èb ø èa ø
(c) log (ab ) (d) None of these
JEE Type Solved Examples :
Single Option Correct Type Questions

x sin (sin x ) - sin 2 x 2 tan K 1x


× K 1x
l Ex. 1 The value of lim equals 2 tan K 1x K 1x
x ®0 x 6
Þ lim = 1 Þ lim =1
x ® 0 tan K 2 x x ® 0 tan K 2 x
1 1 × K 2x
(a) (b) K 2x
6 12
2K 1
(c)
1
(d)
1 Þ = 1 Þ 2K 1 = K 2
18 24 K2
x sin (sin x ) - sin 2 x 1
Sol. (c) Here, lim l Ex. 3 Let f (q ) = {(1 + tan q ) 3 + ( 2 + tan q ) 3 + ...
x ®0 x6 tan q 2

sin -1(t ) × sin t - t 2 + (10 + tan q ) 3 } - 10 tan q.


= lim -1 6
[put sin x = t ]
t ®0 (sin t ) Then, lim f (q ) is equal to
sin -1
(t ) × sin t - t 2 t6 p-

= lim ×
(sin -1 t )6
2
t ®0 t6
(a) 170 (b) 166 (c) 165 (d) None of these
ì t 3 9 t 5 5t 7 ü ì t3 t5 ü Sol. (c) Here,
ít + + + + Ký × ít - + - Ký - t 2 (1 + tan q )3 1 + 3 tan q + 3 tan 2 q + tan 3 q
6 120 112 þ î 3! 5! = lim
= lim î þ lim
´1 p- tan q2
p- tan 2 q
t ®0 t6 q®
2

2
æ1 1 9 ö
t6ç - + ÷ + higher powers to t [(a + b )3 = a 3 + b 3 + 3a 2b + 3ab 2 ]
è 5! 6 × (3!) 120 ø
= lim æ 3 tan q + 1 ö
t ®0 t6 = lim 3 + tan q + ç ÷ = lim - (3 + tan q )
p - è tan 2 q ø p
1 9 1 1 q® q®
= + - = 2 2
120 120 36 18 é ù
êas lim æ 3 tan q + 1 ö = 0ú
ç ÷
æp ö ê p- è tan q ø
2 ú
log e cot ç - K 1 x ÷ êë 0 ® 2 úû
è4 ø
l Ex. 2 If lim = 1 , then
x ®0 tan K 2 x Þ lim f (q ) = lim [(3 + tan q ) + (3(2) + tan q )+K
p- p-
q® q®
(a) K 1 = K 2 (b) 2K 1 = K 2 2 2
+ (3(10) + tan q )] -10 lim tan q
(c) K 1 = 2K 2 (d) K 1 = 4K 2 p-

2
é æp öù
loge ê cot ç - K 1x ÷ ú = lim 3(1 +2 +3 +... +10) +10 × lim (tan q ) - 10 × lim (tan q )
ë è 4 øû p- p- p-
Sol. (b) Here, lim =1 q®
2

2

2
x ® 0 tan K 2 x
3 ´ 10 ´ 11
= = 165
é æp ö ù 2
log ê cot ç - K 1x ÷ - 1 + 1ú
ë è 4 ø û =1
Þ lim
x ®0 tan K 2 x l Ex. 4 For positive integers K =1, 2 , 3 , ..., n. Let Sk denotes
æ 2 tan K 1x ö the area of DOABK (where ‘O’ is the origin) such that
log ç1 + ÷ Kp
è 1 - tan K 1x ø ÐAOBK = , OA = 1 and OBK = K . The value of the
Þ lim =1 2n
x ®0 tan K 2 x n
× å SK is
1
æ 2 tan K 1x ö 2 tan K 1x lim
n ®¥ n2
log ç1 + ÷ K =1
è 1 - tan K 1x ø 1 - tan K 1x
Þ lim × =1 2 4 8 1
x ®0 2 tan K 1x tan K 2 x (a) (b) (c) (d)
1 - tan K 1x p 2
p 2
p 2
2p 2
Chap 05 Limits 273

Kp p p p
Sol. (a) Here, OB K = K and ÐAOB K = Let M = cos × cos 4 K cos n - 1
2n 23 2 2
1 æ Kp ö é 1 ù æ p ö æpö
\ SK = × (1) ( K ) sin ç ÷ using, D = ab sinq ú sin ç2n - 3 × n - 1` ÷ sin ç 2 ÷
2 è 2n ø êë 2 û =
è 2 ø
=
è2 ø
æ p ö æ p ö
Y 2n - 3 × sin ç n - 1 ÷ 2n - 3 × sin ç n - 1 ÷
BK è2 ø è2 ø
Kp 1
2n 3
K
\ Sn = lim 2 ´ ì2n - 3 × sin p ü ´ 1
í n - 1ý
A n ®¥
æ 1 ö
3
î 2 þ æpö
ç ÷ cosn ç n ÷
1 è 2ø è2 ø
X
O 3
æ p ö
1 K æ Kp ö ç sin n - 1 ÷ æ p ö 3 1
Then, L = lim × × sin ç ÷ = lim 2 × ç 2 ÷ ×ç ÷ ´
n ® ¥ n2 2 è 2n ø n ®¥
ç p ÷ è 4 ø n æpö
cos ç n ÷
1 K æp K ö 1 1 æp ö è 2n - 1 ø è2 ø
= × lim × sin ç × ÷ = × ò x × sin ç x ÷ dx
2n n ® ¥ n è2 n ø 2 0 è2 ø æpö 1 p
33
= 2ç ÷ × =
éæ 1 ù è 4 ø 1 32
ö
1 ê ç -2 px ÷ 2 1 px ú
= ê ç × x × cos
2 êç1 p4 2 ÷ +p ò0 cos 2 × dx ú x + 7 - 3 2x - 3
4244 3÷ ú l Ex. 6 The value of lim is
êë è 0 ø0 úû x ®2 3 x + 6 - 23 3 x - 5
1 é 2 2 æ px ö ù
1 33 34 54
2
= ê × × çsin ÷ ú= 2 (a) (b) (c) (d) None of these
2 êp p è ø
2 0ú p 23 23 25
ë û
x + 7 - 3 2x - 3 é0 ù
Sol. (b) We have, L = lim
l Ex. 5 If x ®2 3 x + 6 - 2 3 3x - 5 êë 0 form úû
æ pöæ pö æ pö x - 2 = t , such that x ® 2 Þ t ® 0
Sn = ç1 - tan 4 ÷ ç1 - tan 4 K ç1 - tan 4
4÷ ÷. Let
è 3ø è ø è 2n ø
2 2 (t + 9 )1/ 2 - 3(2t + 1)1/ 2
The value of lim Sn , is \ L = lim
n ®¥ (t + 8)1/ 3 - 2(3t + 1)1/ 3
t ®0
p 3
p 3
(1 + t /9 )1/ 2 - (2t + 1)1/ 2
3 é0 ù
(a) (b) = × lim êë 0 form úû
4 16 2 t ®0 æ t ö1/ 3
ç1 + ÷ - (1 + 3t )
1/ 3
p 3
p3
(c) (d) è 8ø
32 256
æ 1 tö æ 1 ö
Sol. (c) Here, ç1 + × ÷ - ç1 + × (2t )÷
3 è 2 9ø è 2 ø
æ pöæ pö æ pö = × lim ;
Sn = ç1 - tan 4 3 ÷ ç1 - tan 4 4 ÷ K ç1 - tan 4 n ÷ 2 t ®0 æ 1 t ö æ 1 ö
è 2 ø è 2 ø è 2 ø ç 1 + × -
÷ ç 1 + × ( 3t ) ÷
è 3 8ø è 3 ø
æ 2 p 2 p ö æ 2 p 2 p ö
ç cos 3 - sin 3 ÷ ç cos 4 - sin 4 ÷ [using (1 + x )n = 1 + nx , as x ® 0]
è 2 2 øè 2 2 ø
p pö t æ1 ö
æ -t ç - 1÷
K ç cos 2 n - sin 2 n ÷ 3 3 è 18 ø 34
è 2 2 ø = × lim 18 = =
= 2 t ®0 t - t 2 æ 1 ö 23
æ 4 p 4 p 4 p ö ç - 1÷
ç cos 3 × cos 4 K cos n ÷ 24 è 24 ø
è 2 2 2 ø
æ pö æ pö æ p ö f (x + a ) f ( x + 2a ) f ( x + 3a )
ç cos 2 ÷ × ç cos 3 ÷K ç cos n - 1 ÷
è 2 ø è 2 ø è 2 ø
= l Ex. 7 Let D ( x ) = f (a ) f ( 2a ) f (3a ) for
æ p p p ö
ç cos 3 × cos 4 K cos n ÷
4 4 4
è 2 2 2 ø f ¢ (a ) f ¢ ( 2a ) f ¢ (3a )
1 some real values differential function f and constant a,
2 1 D (x )
= × …(i) lim is equal to
4 p
3 x ®0 x
æ p p p ö
ç cos 3 × cos 3 K cos n - 1 ÷ cos 2n
è 2 2 2 ø (a) 0 (b) 1 (c) 2 (d) 3
274 Textbook of Differential Calculus

f (a ) f (2a ) f (3a ) (1 + h )a - a(1 + h ) + a - 1


lim
Sol. (a) Here, D (0) = f (a ) f (2a ) f (3a ) h ®0 h2
f ¢ (a ) f ¢ (2a ) f ¢ (3a ) æ a(a - 1) 2 ö
ç1 + ah + h + ...÷ - a - ah + a - 1
è 2! ø
D(x ) é0 ù = lim
Thus, lim êë 0 form úû h ®0 h2
x ®0 x
a(a - 1)
D ¢( x ) \ f (a ) = ; f (101) = 5050
= lim = D ¢ ( 0) [applying L’Hospital’s rule] 2
x ®0 1
D(x ) nx n +1 - (n + 1) x n + 1
\ lim = D ¢ ( 0) ...(i) l Ex. 9 lim , where n =100 is equal to
x ®0 x x ®1 (e x - e ) sin px
f (x + a ) f ( x + 2a ) f ( x + 3a )
5050 100 5050 4950
Given, D( x ) = f (a ) f ( 2a ) f ( 3a ) (a) (b) (c) - (d) -
pe pe pe pe
f ¢ (a ) f ¢ ( 2a ) f ¢(3a )
nx n ( x - 1) - ( x n - 1)
Using definition of differentiation of determinant, Sol. (c) Let l = lim
x ®1 (e x - e ) sin px
f ¢ ( x + a ) f ¢ ( x + 2a ) f ¢ ( x + 3a )
Put x = 1 + h , so that as x ® 1, h ® 0
D¢ ( x ) = f (a ) f ( 2a ) f ( 3a )
h × n (1 + h )n - {(1 + h )n - 1}
f ¢ (a ) f ¢ ( 2a ) f ¢ ( 3a ) \ l = - lim
h ®0 e (e h - 1) sin ph
f (x + a ) f ( x + 2a ) f ( x + 3a )
Þ l = - lim
+ 0 0 0 x ®1

f ¢ (a ) f ¢ ( 2a ) f ¢ ( 3a ) n × h (1 + n C 1h + n C 2h 2 + n C 3h 3 + ...) - (1 + n C 1h
f (x + a ) f ( x + 2a ) f ( x + 3a ) + n C 2h 2 + n C 3h 3 + K-1)
+ f (a ) f ( 2a ) f ( 3a ) æ e h - 1 ö æ sin ph ö
pe ( h 2 ) ç ÷ç ÷
0 0 0 è h ø è ph ø

[as a is constant Þ
d
(a ) = 0] n 2 - nC 2 é 2n 2 - n (n - 1) ù n2 + n n ( n + 1)
dx =- = -ê ú=- =-
pe ë 2p e û 2( pe ) 2( pe )
f ¢( x + a ) f ¢ ( x + 2a ) f ¢ ( x + 3a )
æ 5050 ö
\ D¢ ( x ) = f (a ) f ( 2a ) f ( 3a ) If n = 100 Þ l = - ç ÷
è pe ø
f ¢ (a ) f ¢ ( 2a ) f ¢ ( 3a )
f ¢ (a ) f ¢ ( 2a ) f ¢ ( 3a ) 1 2 n + 2 2 (n - 1) + 3 2 (n - 2 ) + ... + n 2 × 1
l Ex. 10 lim is
or D¢ (0) = f (a ) f ( 2a ) f ( 3a ) = 0 n ®¥ 13 + 2 3 + 3 3 + ... + n 3
f ¢ (a ) f ¢ ( 2a ) f ¢ ( 3a ) equal to
[Q R1 and R 3 are identical] 1 2 1 1
D(x ) (a) (b) (c) (d)
Hence, lim = D ¢ ( 0) = 0 3 3 2 6
x ®0 x Sol. (a) We have,
x a - ax + a - 1 12 n + 22 (n - 1) + 32 (n - 2) + ... + n 2 {n - (n - 1)}
l Ex. 8 Let lim = f (a ). The value of f (101) lim
n ®¥ Sn 3
x ®1 ( x - 1) 2
Numerator = n (12 + 22 + ... + n 2 )
equals
- {1 × 22 + 2 × 32 + 3 × 4 2 + ... + (n - 1)n 2 }
(a) 5050
n
(b) 5151 = n Sn 2 - S ( r - 1 ) × r 2
r =2
(c) 4950
n
= n Sn 2 - S ( r 3 - r 2 )
(d) 101
r =1
x a - ax + a - 1
Sol. (a) We have, lim
( x - 1)
x ®1 2 = n S n - [ S n 3 - S n 2 ] = ( n + 1 ) Sn 2 - S n 3
2

Put x = 1 + h , we have
Chap 05 Limits 275

æ b (bt - 1) - a (at - 1) ö
( n + 1 ) Sn 2 - Sn 3 lim ç ÷ b ln b - a ln a
\ l = lim t ® 0 çè t (b - a ) ÷
ø b-a
n ®¥ Sn 3
=e =e
4 (n + 1) n (n + 1) (2n + 1) 4 1
= lim - 1= - 1 = b
b 1
n ®¥ 6n (n + 1) n (n + 1) 3 3 ln bb - ln aa
ln
a a æ bb ö b - a
ln çç ÷÷
1

b-a b-a è aa ø æ bb ö b - a
12 n + 22 (n - 1) + ...+ n 2 × 1 =e =e =e = ç a÷
Aliter l = +1-1 èa ø
13 + 23 + 33 +...+ n 3
14442444 3
cot -1 ( x + 1 - x )
12 (n + 1) + 22 (n + 1) +...+ n 2 (n + 1) l Ex. 13 lim is equal to
l= -1 x ®¥ ìïæ 2 x + 1 ö x üï
Sn 3
sec -1 íç ÷ ý
( n + 1 ) Sn 2
l= -1 ïîè x - 1 ø ïþ
Sn 3
(n + 1) × n (n + 1) (2n + 1) 4 1 p
Þ lim -1= -1= (a) 1 (b) 0 (c) (d) None of these
n ®¥ n ( n + 1)
2 2
3 3 2
p
6× Sol. (a) As, lim x + 1 - x = 0 Þ cot -1 (0) =
4 x ®¥ 2
-1 -a
cot (x log a x ) æ 2x + 1 ö
x
p
l Ex. 11 The value of lim (a > 1) is Þ sec -1 ( ¥ ) =
lim ç ÷ ®¥
x ®¥ sec - 1 (a x log x a ) x ®¥ è x - 1 ø 2
equal to \ l =1
p
(a) 1 (b) 0 (c) (d) Doesn’t exist
2 l Ex. 14 lim cos ( p n 2 + n ) (when n is an integer) is
n ®¥
æ loga x ö
cot -1 ç a ÷ equal to
è x ø
Sol. (a) We have, lim ;
æ ax ö
x ®¥ (a) 1 (b) - 1 (c) 0 (d) Doesn’t exist
-1
sec ç ÷
è loga x ø Sol. (c) Let l = lim ± cos (np - p n + n ) 2
n ®¥
æ log x ö æ ax ö = lim ± cos ( p (n - n 2 + n ))
as lim ç aa ÷ ® 0 and ç ÷®¥ n ®¥
x ®¥ è x ø è loga x ø
æ ( p ( + n )) ö
[using L’ Hospital’s rule] = lim ± cos ç ÷
n ®¥ çn + n2 + n ÷
p è ø
\ l = 2 =1 æ ö
p ç ÷
np
2 = lim ± cos ç ÷
n ®¥ ç 1÷
çn + n 1 + ÷
l Ex. 12 Suppose that a and b (b ¹ a ) are real positive è nø
1/t æ ö
æ bt + 1 - at + 1 ö ç ÷
numbers, thene the value of lim ç ÷ is equal p p
t ®0 è b -a = lim ± cos ç ÷ = cos ® 0
ø n ®¥ ç 1 ÷ 2
ç1 + 1 + ÷
to è n ø
a ln b - b ln a b ln b - a ln a Aliter
(a) (b)
b -a b -a
We have, p n 2 + n
1
1/ 2
æ bb öb - a æ 1ö é 1 1 æ1 ö1 1 ù
(c) b ln b - a lna (d) ç a ÷ = pn ç1 + ÷ = np ê1 + + ç - 1÷ + ...ú
èa ø è nø ë 2n 2 è 2 ø 2 ! n 2
û
1 é 1 1 æ1 ö1 1 ù
æ bt +1 - at +1 ö t = p ên + + ç - 1÷ + ...ú
Sol. (d) Here, lim ç ÷ . ë 2 2 è 2 ø 2 ! n û
t ®0 è b -a ø
p é æ1 ö 1 1 ù p
As n ® ¥; × ê2n + 1 + ç - 1÷ + ...ú = (2n + 1)
Obviously, limit is of the form 1¥ . 2 ë è2 ø 2! n û 2
1 é bt + 1 - at + 1 ù æ bt + 1 - at + 1 - b + a ö
lim ê - 1ú lim ç
t ® 0 çè
÷
÷ æ pö
Hence, l = e
t ® 0 t êë b-a úû
=e
t (b - a ) ø \ lim cos ç(2n + 1) ÷ = 0
n ®¥ è 2ø
276 Textbook of Differential Calculus

(tan({ x } - 1)) sin { x } 1 - cos ax sin 2 ax 1


l Ex. 15 The value of lim , where { x } Now, - lim = - lim ×
x ®0 { x } ({ x } - 1) x ®0 sin bx2 x ®0 sin bx 1 + cos ax
2

denotes the fractional part function is a2


-
a2 2b 2
(a) 1 (b) tan 1 (c) sin 1 (d) Doesn’t exist =- , l =e
2b 2
(tan({ x } - 1)) sin { x }
Sol. (d) Let f ( x ) = lim n
r
x ®0 { x } ({ x } - 1) l Ex. 19 lim
n ®¥
å n 2 + n + r equals
tan (h - 1) × sin h tan ( - 1) r =1
lim f ( x ) = lim = = tan1
x ®0 +
h ®0 + h ( h - 1) -1 (a) 0 (b) 1/3 (c) 1/2 (d) 1
tan ((1 - h ) - 1) sin (1 - h ) sin 1 1 2 n
LHL = lim = = sin 1 Sol. (c) Let f (n ) = + + ... +
h ®0 ( 1 - h ) ( 1 - h - 1) 1 n2 + n + 1 n2 + n + 2 n2 + n + n
Hence, lim f ( x ) doesn’t exist. 1 2 n
x ®0 Consider g (n ) = + + ... +
n +n +n
2
n +n +n
2
n +n +n
2

Ex. 16 lim ( - ln({ x } + |[ x ]| )) {x} 1 + 2 + 3 + ...+ n n ( n + 1)


l is equal to = =
x ® 0- n 2 + 2n 2 (n 2 + 2n )

(a) 0 (b) 1 (c) ln2 (d) ln


1 g (n ) < f (n ) ...(i)
2 1 2 n
Similarly, h (n ) = 2 + + ...+ 2
Sol. (d) We have, lim = ( - ln ({ x } + |[ x ]| )){ x } n + n + 1 n2 + n + 1 n +n +1
x ®0 -
n ( n + 1)
= lim ( - ln ({0 - h } + |[ - h ]| )){ - h } =
x ® 0- 2(n 2 + n + 1)
æ1ö \ f (n ) < h (n ) …(ii)
= lim ( - ln (1 - h + 1))1 - h = - ln 2 = ln ç ÷
x ®0 - è2ø From Eqs. (i) and (ii), g (n ) < f (n ) < h (n )
1
2 + 2 x + sin 2 x But lim g (n ) = lim h (n ) = ×
l Ex. 17 lim is equal to n ®¥ n ®¥ 2
x ®¥ ( 2 x + sin 2 x ) e sin x 1
Hence, using Sandwich theorem, lim f (n ) = .
(a) 0 (b) 1 (c) - 1 (d) Non-existent n ®¥ 2
2 + 2x + sin 2x
Sol. (d) We have, lim
+ sin 2x )e sin x
x ®¥ (2x l Ex. 20 The value of lim ( n 2 + n + 1 - [ n 2 + n + 1 ]),
n ®¥
2 sin 2x
+2+ where [×] denotes the greatest integer function is
= lim x x , as x ® ¥
x ®¥ æ sin 2x ö sin x (a) 0 (b) 1/2 (c) 2/3 (d) 1/4
ç2 + ÷e
è x ø Sol. (b) We know that, n < n + n + 1 < n + 1. 2

2 1 [ n2 + n + 1] = n
Þ l = lim = oscillatory between and Hence,
x ®¥ 2 × e sin x e
n +1 1
1 \ l = lim ( n 2 + n + 1 - n ) = lim =
Þ Non-existent n ®¥ n ®¥ n +n +1 +n
2 2
e -1
2
l Ex. 18 The value of lim (cos ax ) cosec bx
is sin 2 ( x 3 + x 2 + x - 3 )
x ®0 l Ex. 21 lim has the value equal to
æ 8b 2 ö æ 8a 2 ö
x ®1 1 - cos ( x 2 - 4 x + 3 )
ç- ÷ ç- ÷
ç ÷ ç ÷
è a2 ø è b2 ø (a) 18 (b) 9/2 (c) 9 (d) None of these
(a) e (b) e
æ a2 ö æ b2 ö sin ( x + x + x - 3)
2 3 2
ç-
ç
÷
÷
ç-
ç
÷
÷
Sol. (a) We have, lim
(c) e
è 2b 2 ø
(d) e
è 2a 2 ø x ®1 1 - cos( x 2 - 4 x + 3)
sin 2 ( x 3 + x 2 + x - 3) ( x 3 + x 2 + x - 3) 2
= lim ×
Sol. (c) Let l = lim(cos ax )cosec
2
bx x ®1 ( x + x + x - 3)
3 2 2
1 - cos ( x 2 - 4 x + 3)
x ®0
( x 2 - 4 x + 3) 2 ( x 3 + x 2 + x - 3) 2
lim
x®¥
= (1) lim ×
Þ l =e cosec bx (cos ax - 1)
2 x ®1 1 - cos ( x - 4 x + 3)
2
( x 2 - 4 x + 3) 2
Chap 05 Limits 277

2
æ x 3 + x 2 + x - 3ö (a) 1 (b) 2
= (1) (2) lim ç ÷ = 2l
2
(c) 7 (d) None of these
x ®1 è x 2 - 4x + 3 ø

3x 2 + 2x + 1 log (1 + 7 f ( x )) - sin ( f ( x )) é0 ù
where, l = lim [using L’ Hospital’s rule] Sol. (b) Here, lim êë 0 formúû
x ®1 2x - 4 x ® ea 3f (x )
6
= = - 3 Þ l = 2 ( - 3)2 = 18 7 f ¢ ( x ) - {cos ( f ( x )) × f ¢ ( x )} {1 + 7 f ( x )}
-2 = lim
x ®e a 3 f ¢ ( x ) × {1 + 7 f ( x )}
l Ex. 22 The graph of function y = f ( x ) has a unique
[using L’Hospital’s rule]
tangent at (e a , 0 ) through which the graph passes, then
7 - cos ( f ( x )) {1 + 7 f ( x )} 7 - 1
log (1 + 7 f ( x )) - sin ( f ( x )) = lim = =2
lim equals x ® ea 3{1 + 7 f ( x )} 3
a
x ®e 3 f (x )

JEE Type Solved Examples :


More than One Correct Option Type Questions
3 x 2 + ax + a + 1 (a) lim f ( x ) = 1 (b) lim f ( x ) = 1
l Ex. 23 If f ( x ) = , which of the following x ® 0+ x ® 0-
x2 +x -2
æ ö
2
can be correct?
(c) cot -1 ç lim f ( x )÷ = 1 (d) None of these
(a) lim f ( x ) exists Þ a = - 2 èx ® 0 - ø
x ®1
ì tan 2 { x }
(b) lim f ( x ) exists Þ a = 13 ï 2 , x >0
ï x - [x ]
x ®- 2 2

(c) lim f ( x ) = 4 / 3 Sol. (a, c) We have, f ( x ) = í 1 , x =0


x ®1
ï { x } cot { x } , x < 0
(d) lim f ( x ) = - 1 /3 ï
x ®- 2
3x 2 + ax + a + 1 î
Sol. (a, b, c, d) Given, f ( x ) =
( x + 2) ( x - 1) tan 2 {h } tan 2 h
RHL = lim f ( x ) = lim = lim =1
As x ® 1, Dr. ® 0, hence as x ® 1, Nr. ® 0 x ® 0+ h ®0 h 2 - [h ]2 h ®0 h2
\ 3 + 2a + 1 = 0 Þ a = - 2 LHL = lim f ( x ) = lim { - h } cot { - h }
x ® 0- h ®0
As x ® - 2, Dr. ® 0, hence as x ® - 2, Nr. ® 0
\ 12 - 2a + a + 1 = 0 Þ a = 13 = lim (1 - h ) cot (1 - h ) = cot1
h ®0
3x - 2x - 1
2
æ ö
2
Now, lim f ( x ) = lim \ cot -1 ç lim f ( x )÷ = cot -1 ( cot 1 )2 = 1
x ®1 x ® 1 ( x + 2) ( x - 1)
èx ® 0- ø
( 3x + 1) ( x - 1) 4
= lim =
x ® 1 ( x + 2) ( x - 1) 3 lEx. 25 Given that the derivative f ¢ (a ) exists. Indicate
3x + 13x + 14
2
( 3x + 7 ) ( x + 2) 1 which of the following statement(s) is/are always true?
and lim = lim =-
x ®- 2 ( x + 2) ( x - 1) x ® - 2 ( x + 2) ( x - 1) 3 f (h ) - f (a )
(a) f ¢ (a ) = lim
h ®a h -a
ì tan 2 { x }
ï 2 , for x > 0 (b) f ¢ (a ) = lim
f (a ) - f (a - h )
ïï x - [ x ] 2
h ®0 h
l Ex. 24 Let f ( x ) = 1 , for x = 0, where [ x ] is
í f (a + 2 t ) - f (a )
ï { x } cot { x } , for x < 0 (c) f ¢ (a ) = lim
t ®0 t
ï
ïî f (a + 2 t ) - f (a + t )
(d) f ¢ (a ) = lim
the step up function and { x } is the fractional part function t ®0 2t
of x, then
278 Textbook of Differential Calculus

Sol. (a, b) Here, options (a) and (b) are true by definition. n
[(r + 1)x + 1)] - (rx + 1)
= lim å
= 0 (rx + 1) [(r + 1) x + 1]
Option (c) is false, as n ®¥
r
f ( a + 2t ) - f ( a )
lim = 2 f ¢ (a ) n
æ 1 1 ö
t ®0 t = lim
n ®¥
å çè rx + 1 - (r + 1) x + 1 ÷ø
f ( a + 2t ) - f ( a + t ) 1 1 r =0
and lim = [2 f ¢ (a ) - f ¢ (a )] = f ¢ (a )
t ®0 2t 2 2 éæ1 1 ö æ 1 1 ö ù
Hence, option (d) is false. êç - ÷+ç - ÷ ú
è 1 x + 1 ø è x + 1 2x + 1 ø
= lim ê ú
n n ®¥ ê æ 1 1 ö ú
x ê + ...+ ç -
l Ex. 26 Let f ( x ) = lim
n ®¥
å (rx + 1) {(r + 1) x + 1} . Then, ë è nx + 1 ( n + 1 ) x + 1
÷
ø
ú
û
r =0
é 1 ù ( n + 1) x
(a) f (0) = 0 (b) f (0) = x = lim ê1 - = lim
n ®¥ ë (n + 1)x + 1 úû n ® ¥ (n + 1)x + 1
(c) f (0+ ) = 1 (d) f (0- ) = 1
ì0, x = 0
Þ f (0) = 0, f (0+ ) = f (0- ) = 1.
n
x \ f (x ) = í
Sol. (a, c, d) Given, f ( x ) = lim
n ®¥
å (rx + 1) {(r + 1) x + 1} î1, x ¹ 0
r =0

JEE Type Solved Examples :


Passage Based Questions
Passage I n
Sol. (Q. Nos. 27 to 29)
(Q. Nos. 27 to 29) - sin h + tan h + cos h - 1
-0
2h 2 + ln (2 - h ) - tan h
ì 2
e { x } - 1, x >0 LHD = lim
ï h ®0 -h
ï sin x - tan x + cos x - 1 sin h tan h 1 - cos h
Let f ( x ) = í , x < 0, - + ´h
ï 2 x + ln( 2 + x ) + tan x h2
2
= lim h 2 h =0
ïî 0, x =0 h ®0 2h + ln (2 - h ) - tan h
2 2
where { } represents fractional part function. Suppose lines L1 eh - 1 - 0 eh - 1
and L 2 represent tangent and normal to curve y = f ( x ) at f ¢ (0+ ) = RHD = lim =h ´ =0
h ®0 h h2
x = 0 . Consider the family of circles touching both the lines
L1 º y = 0 and L 2 º x = 0
L1 and L 2 .
27. (a) ( x - r )2 + (y - r )2 = r 2 [family of circle]
l Ex.27 Ratio of radii of two circles belonging to this x 2 + y 2 - 2rx - 2ry + r 2 = 0
family cutting each other orthogonally is
Y
(a) 2 + 3 (b) 3
(c) 2 + 2 (d) 2 - 2

l Ex.28 A circle having radius unity is inscribed in the


triangle formed by L1 and L 2 and a tangent to it. Then, the
minimum area of the triangle possible is O X

(a) 3 + 2 (b) 2 + 3
Q 2(r1r 2 + r1r 2 ) = r12 + r 22
(c) 3 + 2 2 (d) 3 - 2 2
or 4r1r 2 = r12 + r 22
l Ex.29 If centres of circles belonging to family having 2
æ r2 ö æ r2 ö
equal radii r are joined, the area of figure formed is ç ÷ - 4 ç ÷ +1=0
è r1 ø è r1 ø
(a) 2r 2 (b) 4r 2
r 2 4 ± 12
(c) 8r 2 (d) r 2 Þ = =2± 3
r1 2
Chap 05 Limits 279

28. (c) 2 [ D 1 + D 2 + D 3 ] Such that each horizontal row is arithmetic progression and
each vertical column is a geometrical progression. It is
1 æ æp qö q ö é 1 ù
D =2´ ç cot ç - ÷ + cot + 1÷ êë using, 2 ab úû known that each column in geometric progression have the
2 è è4 2ø 2 ø
1 3
same common ratio. Given that a 24 = 1, a 42 = and a 43 = .
æp qö
cos ç - ÷ cos q 8 16
è4 2ø 2 +1
D= +
æ p q ö sin q n
S
sin ç - ÷
è4 2ø 2
l Ex.30 Let Sn = å a 4 j , nlim n
®¥ n2
is equal to
j =1
Y 1 1 1 1
p/2 – q (a) (b) (c) (d)
p/4 – q/2 4 8 16 32
Sol. (d) Here, a 43 = a 42 + d
è
r cot è4 – 2 è
p q

D1 3 1 1
Þ = +d Þ d =
è

16 8 16
[common difference of 4th row]
1 1 1
r \ a 41 = a 42 - d = - =
D3
8 16 16
r (r, r) 1 2 3 n
q/2
q \ a 41 = , a 42 = , a 43 = ,...,a 4n =
D2 q/2 16 16 16 16
O X Now, all elements of 4th row are known
r cot (q/2)
n
n ( n + 1) S 1
p
2 sin Sn = å a 4 j = ; lim n2 =
2 ( 16 ) n ®¥ n 32
4 j =1
Þ D =1+
q æp qö
2 sin × sin ç - ÷
2 è4 2ø l Ex.31 Let d i be the common difference of the elements in
n
2
D =1+ ith row, then å d i is
æ pö æpö
cos çq - ÷ - cos ç ÷ i =1
è 4ø è4ø
1 1 1 n +1
p (a) n (b) - (c) 1 - (d)
D is minimum, if denominators is maximum when q = , 2 2n + 1 2n
2n
4
4
2 2 Sol. (c) Also, a 24r 2 = a 44 =
D min = 1 + =1+ = 1 + 2 ( 2 + 1) = 3 + 2 2 16
1-
1 2 -1
4 1 é 1ù
2 Þ r2 = Þ r = êë common ratio of all GP is 2 úû
16 2
29. (b) Area = (2r )2 = 4r 2
Y é1 2 3 nù
ê2 ...
2 2 2ú
ê1 2 3 nú
ê 2 ... 2 ú
2r ê2 22 22 2 ú
\ A=ê1 2 3 n
X' X
... 3 ú
ê 23 23 23 2 ú
ê M M M M Mú
ê1 2 3 nú
Y' êë 2n ... n ú
2n 2n 2 ûn ´ n
Passage II 1
Now, di = common difference in i th row =
(Q. Nos. 30 to 32) 2i
Let A be n ´ n matrix given by 1æ 1ö
ç1 - n ÷
n
2 èn
2 ø =1- 1
1
éa 11 a 12 a 13 K a 1n ù \ ådi = å 2i =
êa a 22 a 23 K a 2n ú i =1 i =1 1-
1 2n
A = ê 21 ú 2
ê M M M ú
ëa n1 a n 2 a n3 ... a nn û
280 Textbook of Differential Calculus

n Y
l Ex.32 The value of lim
n ®¥
å a ii is equal to
i =1
1 1 Case IV
(a) (b) (c) 1 (d) 2 f (a )
4 2
n
1 2 3 n
åaii = S = 2 + 22
X
Sol. (d) Given, + + K+ …(i) O a
i =1 23 2n
1 1 2 3 n -1 n If f (a ) is an integer, the limit will exist in Case III and
Þ Sn = 2 + 3 + 4 + ...+ n + n + 1 …(ii)
2 2 2 2 2 2 Case IV but not in Case I and Case II. If f (a ) is not an
On subtracting Eq. (ii) from Eq. (i), we get integer, the limit exists in all the cases.
1 æ1 1 1 1ö n
Sn = ç + 2 + 3 + K+ n ÷ - n + 1 é 1ù
2 è 2 2 2 2 ø 2 l Ex.33 If f ¢ (1) = - 3 and lim ê f ( x ) - ú does not exist,
x ®1 ë 2û
1æ 1ö
ç1 - n ÷ (where [× ] denotes the greatest integer function), then
1 2è 2 ø - n Þ S = 2æ1 - 1 ö - n
Þ Sn = ç ÷
è 2n ø 2n
n
2 1-
1 2n + 1 (a) f (1) may be integer
2 1
n
(b) { f ( x )} = , " x Î R ({×} fractional part of x )
2 2n
\ lim å aii = lim 2 - n - n = 2 - 0 - 0 = 2
2
n ®¥ n ®¥ 2 2 (c) f (1) is not an integer
i =1
n (d) None of the above
Þ lim
n ®¥
åaii = 2 é
Sol. (c) lim f ( x ) -

does not exist, when f (1) is not an
x ®1 ê 2 úû
i =1
ë
Passage III integer, as f ¢ (1) = - 3, i.e. decreasing in the neighbour-
hood at x = 1.
(Q. Nos. 33 to 35)
To evaluate lim [ f (x )], we must analyse the f ( x ) in right é sin x ù
x®a l Ex.34 lim ê(1 - e x ) × (where [ × ] denotes the great-
hand neighbourhood as well as in left hand neighbourhood x ®0 ë | x | úû
of x = a. e.g. In case of continuous function, we may come est integer function), equals
across following cases.
(a) 0 (b) 1 (c) -1 (d) Doesn’t exist
Y
ì sin x
ï (1 - e ) × x , x > 0
x

Sol. (c) Let f ( x ) = í .


sin x
Case I f(a) ï ( e x - 1) × , x <0
î x
ì < 0, for x > 0
\ f (x ) = í Þ lim [ f ( x )] = - 1
X
î < 0, for x < 0 x ®0
O a
-1/(1- x )
Y é px ù
l Ex.35 lim êcosec ú is equal to (where [× ]
x ®1 ë 2û
denotes the greatest integer function).
Case II
(a) 0 (b) 1 (c) ¥ (d) Doesn’t exist
f(a)
æ px ö
Sol. (b) Let f ( x ) = cosec ç ÷
X è 2 ø
O a
æp ö
Y Þ f (1+ ) = lim cosec ç (1 + h )÷ > 1
h ®0 è2 ø
æp ö
and f (1- ) = lim cosec ç (1 - h )÷ > 1
Case III h ®0 è2 ø
f(a)
-1
é px ù é px ù 1 - x
a X \ lim ê cosec ú = 1 Þ lim ê cosec ú =1
O x ®1 ë 2 û x ®1 ë 2 û
JEE Type Solved Examples :
Statements I and II Type Questions
n Directions (Ex. Nos. 36 to 38) This section is based on l Ex. 37 Statement I lim lim {sin 2m n ! pn } = 0 m, n Î N ,
Statements I and II. Select the correct answer from the m ®¥n ®¥
codes given below. when x is rational.
(a) Statement I is true, Statement II is true; Statement II Statement II When n ® ¥ and x is rational n ! x , is integer.
is correct explanation for Statement I
b
(b) Statement I is true, Statement II is true; Statement II Sol. (a) When n ® ¥ and x is rational x = .
is not the correct explanation for Statement I q
(c) Statement I is true, Statement II is false p
Where p and q are integers and q ¹ 0 n ! x = n !´ is integer
(d) Statement I is false, Statement II is true q
as n ! has factor q, when n ® ¥
l Ex. 36 Also, when n ! x is integer, sin(n ! p x ) = 0.
sin(cot 2 x ) 1 Therefore, the given limit is zero.
Statement I lim = .
(p - 2x )
x ® p /2 2 2
ì sin x ü
sin q tan q l Ex. 38 Statement I If lim í f ( x ) + ý does not exist,
Statement II lim = 1 and lim = 1 , where q x ®0î x þ
q®0 q q®0 q
is measured in radians. then lim f ( x ) does not exist.
x ®0
æ e 1/ x - 1 ö
Statement II lim ç ÷ does not exist.
sin (cot 2 x ) cot 2 x p x ® 0 è e 1/ x + 1 ø
Sol. (d) lim × ; put x = -h
x ® p/ 2 cot x 2
( p - 2x ) 2
2 æ sin x ö
Sol. (b) If lim f ( x ) exist, then lim ç f ( x ) + ÷ always exist as
tan 2 h 1 x ®0 x ®0 è x ø
Þ lim =
h ® 0 4h 2 4 sin x
lim exists finitely.
\ Statement I is false and Statement II is true. x ®0 x
Hence, lim f ( x ) must not exist.
x ®0

JEE Type Solved Examples :


Matching Type Questions
1- y -1
l Ex. 39 Match the following. 1 æ 1 ö
1/ y lim
y ® 0 y (1 + y )
(A) Put x = , lim ç ÷ =e = e -1
Column I Column II y y ® 0 è1 + y ø
æ x ö
x
(p) e2 lim
sin y + cos y - 1
(A) lim ç ÷ equals y®0
x ® ¥ è1 + xø (B) lim (sin y + cos y ) 1/ y
=e y
=e
y ®0
x -1/ 2
æ 1 1ö (q) e
(B) lim ç sin + cos ÷ equals cos x - 1
x®¥ è x xø
lim
x ® 0 tan 2 x 2
×x 1
2 2 -
(C) lim (cos x )cot x
equals (r) e (C) e x =e 2
x®0

é æp öù
1/ x
(s) e-1 lim
tan (( p/ 4 ) + x ) - tan( p/ 4 )
lim tan ç + x÷ ú equals
x®0ê
(D)
ë è4 øû (D) e x ® 0 x

tan x [1 + tan (( p/ 4 ) + x )×1]


Sol. (A) ® (s); (B) ® (r); (C) ® (q); (D) ® (p) lim
x®0
=e x
= e2
282 Textbook of Differential Calculus

l Ex. 40 Match the following. æ sin 3 x ö


(C) lim [ln sin 3 x - ln ( x 4 + ex 3 )] = lim ln ç 3 ÷
x ® 0+ x ®0 è x (x + e )ø
Column I Column II
æ1ö
(A) lim ( x + x - x - x ) equals
(p) -2 = ln ç ÷ = - 1
x®¥
èe ø
sin 2x - 2 tan x (q) -1 æp ö
(B) The value of the lim is (D) tan(2p | sin q | ) = tan ç - | cos q | 2p ÷
x®0 ln (1 + x 3 ) è2 ø
(C) lim (ln sin 3 x - ln(x 4 + ex 3 )) equals (r) 0 p
x ® 0+ Þ 2p |sin q | = n p + - | cos q | 2p
2
(D) Let tan (2p | sin q |) = cot (2p | cos q |), (s) 1
p
where q Î R and f (x ) = (| sin q | + |cos q |) . x Þ 2p (|sin q | + | cos q | ) = np +
2
é 2 ù n 1
The value of lim ê ú equals (here, [×] Þ |sin q | + | cos q | = + …(i)
x ® ¥ ë f (x ) û 2 4
n 1
represents greatest integer function) Since, 1 £ |sin q | + | cos q | £ 2; 1 £ + £ 2
2 4
Sol. (A) ® (s); (B) ® (p); (C) ® (q); (D) ® (r) 4 £ 2n + 1 £ 4 2
(x + x ) - (x - x ) 3 4 2 -1
(A) lim Þ £n £
x ®¥ x+ x + x- x 2 2
2 x Thus, n = 2 is only possible value. Putting in Eq. (i),
= lim =1
x ®¥ x+ x + x- x 5
| sin q | + | cos q | = ,
sin 2x - 2 tan x x3 4
(B) lim lim
x ®0 x3 x ®0 log (1 + x 3 )
é æ 4 öx ù
2 sin x (cos x - 1)
2
2 tan x 3 g ( x ) = lim ê2 ç ÷ ú = 0
= lim × 1 = lim - = -2 x ®¥
êë è 5 ø úû
x ®0 cos x × x 3 x ®0 x3

JEE Type Solved Examples :


Single Integer Answer Type Questions
l Ex. 41 If lim ( x -3 sin 3 x + ax -2 + b ) exists and is equal l Ex. 42 For a certain value of c,
x ®0
lim [( x 5 + 7 x 4 + 2 ) c - x ] = l, is finite and non-zero.
to zero, the value of a + 2b is ……… . x ®-¥

sin 3 x a sin 3x + ax + bx 3 The value of 3c + l is ……… .


Sol. (6) We have, lim + + b = lim
x ®0 x 3
x 2 x ®0 x 3 Sol. (2) We have, lim [( x 5 + 7 x 4 + 2)c - x ]
x ® –¥
sin 3x æ c ö
3 + a + bx 2 æ 7 2 ö
3 x = lim çç x 5c ç1 + + 5 ÷ - x ÷÷
= lim for x ®- ¥
è è x x ø ø
x ®0 x2
existence of limit 3 + a = 0 Þ a = - 3 æ æ 7 2 ö
c ö
= lim x çç x 5c - 1 ç1 + + 5 ÷ - 1÷÷
sin 3x - 3x + bx 3 x ®- ¥
è è x x ø ø
\ l = lim
x ®0 x3 This will be of the form ¥ ´ 0 only, if
sint - t 1 1
= 27 × +b=0 [Q3x = t ] 5c - 1 = 0 Þ c = substituting c = , l becomes
t3 5 5
7 2
27 9 l = lim x [(1 + x )1/ 5 - 1], where x = + 5
=- +b=0 Þ b= x ®- ¥ x x
6 2 é x ù æ7 2 ö 1 7
= lim x ê1 + + ....- 1ú = lim x ç + 5 ÷ × =
Hence,
9
a + 2b = - 3 + 2 ´ = 6 x ®- ¥ ë 5 û x ® - ¥ è x x ø 5 5
2 1 7
[using L’ Hospital’s rule] Hence, c = and l = Þ 3c + l = 2
5 5
Chap 05 Limits 283

Ex. 43 Consider the curve y = tan -1 x and a point x 2 t dt


ò
l 1
l Ex. 44 If lim = ,
x ®0 0
æ pö (e x - 1 - x )
2a t
- + 104
19
A ç1 , ÷ on it. If the variable point Pi ( x i , y i ) moves on the
è 4ø 3 2
a
curve for i = 1 , 2 , 3 , K, n (n Î N ) such that then equals ........... .
r
æ 1 ö
2010
y r = å tan -1 ç ÷ and B ( x , y ) be the limiting position x 2t
m =1
è 2m 2 ø ò0 2a t
dt
- + 104
of variable point Pn as n ® ¥, the value of Sol. (1) Here, lim 3 2 =
1
reciprocal of the slope of AB will be …......... . x ®0 e -1- x
x
19
[using L’Hospital’s rule]
n
æ 1 ö
Sol. (2) Here, y = lim yn = lim
n ®¥ n ®¥
å tan -1 çè 2m 2 ÷ø Þ lim
2x
=
1
m =1
x ®0 2a x 19
- + 104 (e x - 1)
n
æ 2 ö
= lim
n ®¥
å tan -1 çè 1 + (2m + 1) (2m - 1) ÷ø 3 2
m =1 1 x 1
Þ 2 lim × lim x =
n
é (2m + 1) - (2m - 1) ù x ®0 2a x x ® 0 e - 1 19
= lim
n ®¥
å tan -1 ê 1 + (2m + 1)(2m - 1) ú 3
- + 104
2
m =1 ë û
n 1 1
Þ 2× ×1 =
= lim
n ®¥
å (tan -1(2m + 1) - tan -1(2m - 1)) 2a
+ 104
19
m =1
3
= lim (tan -1 3 - tan -1 1) + (tan -1 5 - tan -1 3) 2a a
n ®¥
Þ + 104 = 1444 Þ 2a = 4020 Þ =1
3 2010
+ ...+ (tan -1(2n + 1) - tan -1 (2n - 1))
= lim (tan -1 (2n + 1) - tan -1(1)) l Ex. 45 Evaluate lim log sin x sin 2 x .
n ®¥
x ® 0+
p p p
= - = log sin 2x
2 4 4 Sol. (1) Here, lim log sin x sin 2x = lim
x ® 0+ x ® 0+ logsin x
é pö é–¥ ù
Q B ® ê1, ÷ , i.e. coordinates of B approaches towards
ë 4ø ê – ¥ form ú
ë û
those of A. 1
× 2 cos 2x
Chord AB approaches to the tangent to y = f ( x ) at A. = lim
sin 2x
[using L’Hospital’s rule]
x ® 0+ 1
æd ö × cos x
\ Slope of AB = ç tan -1 x ÷ sin x
è dx ø at x =1
é ( 2x ) ù
æ 1 ö 1 ê sin(2x ) ú cos 2x
=ç ÷ = = lim ë û = lim
cos 2x
=1
è 1 + x 2 ø at x =1
2
x ® 0+ æ x ö x ® 0 + cos x
ç ÷ cos x
Þ (Slope of AB)-1 = 2 è sin x ø
é sin x ù
êëQ xlim = 1ú
®0 x û
Subjective Type Questions
l Ex. 46 Evaluate a , b, c and d, if which shows (n ! px ) is not an integral multiple of p and so,
cos (n ! px ) will lie between –1 and +1.
lim ( x 4 + ax 3 + 3 x 2 + bx + 2
x ® ¥ Thus, lim lim [1 + cos 2m (n ! px )]
m ®¥ n ®¥
– x 4 + 2 x 3 – cx 2 + 3 x – d ) = 4.
= lim lim [1 + (a value between - 1 and + 1)2m ]
m ®¥n ®¥
Sol. Here, lim ( x + ax + 3x + bx + 2
4 3 2
x ®¥
= lim [1 + 0] = 1 [as 0 < x < 1 Þ x ¥ = 0]
n ®¥
– x 4 + 2x 3 – cx 2 + 3x – d ) = 4 [ ¥ – ¥ form]
ì2, x ÎQ
lim [1 + cos (n ! px )] = í
2m
Rationalising Thus, lim
m ®¥ n ®¥ î1, x ÏQ
( a – 2) x 3 + ( 3 + c ) x 2 + ( b – 3) x + ( 2 + d )
lim =4
x ®¥ é x 4 + ax 3 + 3x 2 + bx + 2 ù 4 + 3a n
ê ú
l Ex. 48 If a 1 = 1 and a n + 1 = , n ³1 , show that
ê + x 4 + 2x 3 – cx 2 + 3x – d ú 3 + 2a n
ë û
a n + 2 > a n + 1 and if a n has a limit l as n ® ¥, then evaluate
Since, limit is finite, the degree of the numerator must be 2.
So, a – 2 = 0, i.e. a = 2. lim a n .
n ®¥
( 3 + c ) x 2 + ( b – 3) x + ( 2 + d ) a1 = 1
Þ lim =4 Sol Here,
x ®¥ é
x 4 + ax 3 + 3x 2 + bx + 2 ù 4 +3 7
ê ú \ a2 = = >1
êë + x 4 + 2x 3 – cx 2 + 3x – d úû 3 +2 5
\ a 2 > a1
On dividing numerator and denominator by x 2 , we get
Assuming an + 1 > an
(3 + c ) + (b – 3)/x + (2 + d )/x 2
lim 4 + 3an + 1 4 + 3an
x ®¥ a 3 b 2 2 c 3 d \ an + 2 – an + 1 = –
1 + + 2 + 3 + 4 + 1 + – 2 + 3– 4 3 + 2an + 1 3 + 2an
x x x x x x x x
3+c ( 4 + 3an + 1 ) (3 + 2an ) – ( 4 + 3an ) (3 + 2an + 1 )
=4 Þ =4 =
2 (3 + 2an + 1 ) (3 + 2an )
Þ c =5 an + 1 – an
\ c = 5, a = 2 = >0 [Qan + 1 > an ]
(3 + 2an + 1 ) (3 + 2an )
Hence, a = 2, c = 5 and b, d are any real numbers.
\ an + 2 – an + 1 > 0
l Ex. 47 If x is a real number in [0, 1]. Find the value of Þ an + 2 > an + 1
lim lim [1 + cos (n ! px )].
2m whenever an + 1 > an .
m® ¥ n® ¥
\ The sequence of values an is increasing and since a1 = 1,
Sol. If x is a real number in [0, 1], then we have two cases an > 0, for all n.
either.
Case I x Î Q [rational number] Now, let l = lim an = lim an + 1
n ®¥ n ®¥
1 1 1
As x Î Q , we have x = 0, , , , ..., 1 4 + 3an
2 3 4 \ l = lim
n ®¥ 3 + 2an
which shows (n ! px ) is integral multiple of p for large
values of n. 4 + 3l éQ lim a = l ù
Þ l=
ëê n ® ¥ úû
n
\ cos (n ! px ) = ± 1 3 + 2l
Thus, lim lim [1 + cos 2m (n ! px )] = lim lim (1 + 1) = 2 Þ 3l + 2l 2 = 4 + 3l
m ®¥ n ®¥ m ®¥ n ®¥
Þ l2 = 2
Case II x Ï Q [irrational number]
1 1 1 \ l= 2 [ neglecting l = – 2, as l > 0]
As x Ï Q , we have x = , , , ...
2 3 5
Chap 05 Limits 285

l Ex. 49 Let a 1 , a 2 , K, a n be sequence of real numbers l Ex. 50 A square is inscribed in a circle of radius R, a
with a n + 1 = a n + 1 + a n2 and a 0 = 0. Prove that circle is inscribed in this square then a square in this circle
and so on, n times. Find the limit of the sum of areas of all
æ a ö 4 the squares as n ® ¥.
lim ç n ÷ = .
n ® ¥ è 2n – 1 ø p Sol. Let the side of a square, S1 be ‘a’ units.
Sol. Here, an + 1 = an + 1 + an2 , let an = cot (a n )
R
Þ an + 1 = cot (a n ) + cosec (a n )
cos (a n ) + 1
Þ an + 1 =
sin (a n )
O
2 cos 2 (a n / 2) æa ö
= = cot ç n ÷
2 sin (a n / 2) cos (a n / 2) è 2 ø
S1
Putting n = 1, a1 = cot (a 1 ) C1
and a1 = a 0 + 1 + a 02 = 1
a
p Then, a 2 = 2R Þ R = is radius of circle C 1.
Þ cot (a 1 ) = 1 or a 1 = 2
4
If a1 be the side of another square, then
a
æ ö æpö
Again, a 2 = cot ç 1 ÷ = cot ç ÷ a1 2 = a Þ a1 =
a
è 2 ø è8ø
2
æa ö æ p ö a1 a
a 3 = cot ç 2 ÷ = cot ç ÷ a 2 2 = a1 Þ a 2 = = ×
è 2 ø è 4 × 22 ø 2 2
æa ö æ p ö ................................................
a 4 = cot ç 3 ÷ = cot ç ÷ ................................................
è 2 ø è 4 × 23 ø
................................................ So, sum of areas of all the squares,
................................................ Sn = a 2 + a12 + a 22 + . . . upto n terms
æ p ö a2 a2
an = cot ç ÷; = a2 + + + . . . upto n terms
è 4 × 2n – 1 ø 2 4
ì 1ü
Hence, 1– n ï
æ 1 1 1 ö 2 ï
æ p ö = a ç1 + + + + K upto n terms ÷ = a í 2 ý
2
cot ç ÷ è 2 4 8 ø 1
æ a ö è 4 × 2n – 1 ø 1 ï 1– ï
lim ç n ÷ = lim = lim î 2þ
n ® ¥ è 2n – 1 ø n ®¥ 2n – 1 x ®0 æp ö æ 1ö
tan ç x ÷ = 2a 2 ç1 – n ÷
è4 ø è 2 ø
x æ 1ö
\ lim Sn = lim 2a 2 ç1 – n ÷ = 2 a 2 = 4 R 2
é 1 ù n ®¥ n ®¥ è 2 ø
êë put 2n – 1 = xú
û é 1 ù
\
æ a ö 4
lim ç n n– 1 ÷ = êë as n ® ¥ Þ 2n ® 0úû
n ® ¥ è2 ø p
#L Limits Exercise 1 : Single Option Correct Type Questions
sin ( p cos 2 (tan (sin x ))) 8. Let f ( x ) be a real valued function defined for all x ³ 1,
1. lim is equal to
x ®0 x2 1
satisfying f (1) = 1 and f ¢ ( x ) = ; then
p x + ( f ( x ))
2
(a) p (b)
4
lim f ( x )
p x ®¥
(c) (d) None of these
2 (a) doesn’t exist
1 - ( 1 + t )t p
2. lim is equal to (b) exists and less than
t ® 0 ln (1 + t ) - t 4
p
1 1 (c) exists and less than 1 +
(a) (b) - 4
2 2 (d) exists and equal to 0
(c) 2 (d) - 2
9. The quadratic equation whose roots are the minimum
3. If I 1 = lim (tan -1 px - tan -1 x ) cos x and 1
x ®¥ value of sin 2 q - sin q + and lim ( x + 1) ( x + 2) - x is
2 x ®¥
I 2 = lim (tan -1 px - tan -1 x ) cos x , then ( I 1 , I 2 ) is
x ®0 (a) 3 x 2 - 7 x + 3 = 0 (b) 8 x 2 - 14 x + 3 = 0
(a) ( 0, 0 ) (c) x 2 - 7 x + 3 = 0 (d) 2 x 2 - 7 x + 3 = 0
(b) ( 0, 1 )
xn
(c) (1, 0 ) 10. If x 1 = 3 and x n + 1 = , " n Î N , then
(d) None of the above 1 + 1 + x n2
4. If f ( x ) = 0 is a quadratic equation such that lim 2n x n equal to
x ®¥
æpö 3p 2
3 2 2p 3p
f ( - p ) = f ( p ) = 0 and f ç ÷ = - , (a) (b) (c) (d)
è2ø 4 2p 3p 3 2
f (x ) x -b - a -b
then lim
x ® - p sin (sin x )
is equal to 11. lim- , (a > b ) is
x ®a (x 2 - a 2 )
(a) 0 (b) p 1 1
(c) 2p (d) None of these (a) (b)
4a a a -b
3 1 + sin 2 x - 4 1 - 2 tan x (c)
1
(d)
1
5. lim is equal to 2a a - b 4a a - b
x ®0 sin x + tan 2 x
(a) - 1 (b) 1 12. lim (sin n 1 + cos n 1)n is equal to
1 1 n ®¥
(c) (d) -
2 2 (a) cot 1 (b) tan 1 (c) cos 1 (d) sin 1
1 + xn 2/ x 2
6. If x n + 1 = and | x 0 | < 1, n ³ 0, then æ 2 ö
13. The value of lim ç 3 (sin -1 x - tan -1 x )÷ equals
2 x ®0 èx ø
æ 1 - x 02 ö 1 1
lim ç ÷ is equal to (a) e (b) e (c) (d)
n ® ¥ ç x 1 x 2 x 3 ... x n + 1 ÷ e e
è ø
n
n-K æ 4K ö
(a) - 1
(b) 1
14. The value of lim
n ®¥
å n 2
cos ç
è n ø
÷
k =1
(c) cos-1 ( x 0 + 1 ) equals
(d) cos-1( x 0 ) 1 1 1
(a) sin 4 + cos 4 -
x 4 16 16
7. For n Î N , let f n ( x ) = tan (1 + sec x ) (1 + sec 2x ) 1 1 1
2 (b) sin 4 - cos 4 +
f (x ) 4 16 16
(1 + sec 4 x )... (1 + sec 2n x ). Then, lim n is 1
x ®0 2x (c) (1 - sin 4 )
16
(a) 0 (b) 2n 1
(c) 2n - 1 (d) 2n + 1 (d) (1 - cos 4 )
16
Chap 05 Limits 287

1 - cos 2x × 3 cos 3x × 4 cos 4 x ... n cos nx 23. If f ( x ) = lim


1
, then the set of values
15. If lim has the n ®¥ 2n
x ®0 x 2
æ3 -1 ö
ç tan 2x ÷ +5
value equal to 10, the value of n is èp ø
(a) 6 (b) 7 (c) 8 (d) 9 of x for which f ( x ) = 0, is
z -1 (a) | 2 x | > 3 (b) | 2 x | < 3
16. lim
ò1/ 2 [ cot x ]dx
, where [×] denotes the greatest (c) | 2 x | ³ 3 (d) | 2 x | £ 3
z ®¥ é
z 1ù
ò1/ 2 êë1 + x úû dx 24. The integer ‘n’ for which the
x3
integer function, equals cos 2 x - cos x - e x cos x + e x -
(a) 0 (b) 1 lim 2 is a finite
(c) cot 1 (d) not defined x ®0 xn
17. If a and b are roots of x 2 - ( 1 - cos 2 q ) x + q = 0, where non-zero number, is
(a) 2 (b) 3
p æ 1 1ö
0<q < . Then, lim ç + ÷ is (c) 4 (d) None of these
2 q®0 è a
+ bø
tan -1 x sin -1 x
(a)
1
(b) - 2 25. If I 1 = lim -
x ®0 x x
2
(c) 2 (d) None of these sin -1 x tan -1 x
and I 2 = lim - ,
x ®0
1æ 5 ö x x
18. If f ( x ) = ç f ( x + 1) + ÷ and f ( x ) > 0, " x Î R,
3è f ( x + 2) ø where | x | < 1, which of the following statement is true?
then lim f ( x ) is (a) Neither I 1 nor I 2 exist
x ®¥ (b) I 1 exists and I 2 doesn’t exist
2 5 (c) I 1 doesn’t exist and I 2 exists
(a) 0 (b) (c) (d) ¥
5 2 (d) None of the above

19. Let f :(1, 2) ® R satisfies the inequality éx ù


êë 2 úû
cos (2x + 4 ) - 33 x 2 4x - 8 26. The value of lim is equal to
< f (x ) < , " x Î (1, 2). Then, x ® p/ 2 log (sin x )
2 x -2
(a) 0 (b) 1
lim f ( x ) is (c) -1 (d) doesn’t exist
-
x ®2
(a) 16 (b) -16 1öææ 1ö æ 1ö
27. The value of lim ç1 + ÷ ç1 + ÷ ... ç1 + ÷ , where
(c) 8 (d) doesn’t exist n ®¥ a1 ø è a 2 ø è an ø
è
20. Let f ( x ) be polynomial of degree 4 with roots 1, 2, 3, 4 a1 = 1 and an = n (1 + an – 1 ), " n ³ 2 , is
and leading coefficient 1 and g ( x ) be the polynomial of e
(a) e (b)
1 1 1 2
degree 4 with roots 1, , and with leading coefficient
2 3 4 (c) 2e (d) 3e
f (x ) 28. If f ( x + y ) = f ( x ) + f (y ) for all x , y Î R and f (1) = 1,
1. Then, lim equals
x ®1 g ( x )
2 f (tan x ) – 2 f (sin x )
1 1 1 then lim equals
(a) (b) -24 (c) (d) - x ®0 x 2 f (sin x )
24 12 12
(a) log 2 (b) log 4 (c) log 2 (d) log 8
4 2 - (cos x + sin x ) 5
21. The value of lim é 1 æ 1ö
29. The value of lim n –n ê(n + 1) æçn + ö÷ çn +
is 2

p 1 - sin 2x ÷
4 n ®¥ è 2ø è
ë 22 ø
(a) 2 (b) 3 5 n
(c) 5 2 (d) -5 2 æ 1 öù
... çn + n – 1 ÷ ú is
è 2 øû
n × 3n
1
22. If lim = , where n Î N , the
n +1 (a) e
- 2) + n × 3 - 3
n ®¥ n ( x n 3 n
(b) e 2
number of integer(s) in the range of ‘x’ is (c) e 3
(a) 3 (b) 4 (d) e 4
(c) 5 (d) infinite
288 Textbook of Differential Calculus

ì sin [x ] (a) x 2 (b) x 3 + 2 x 2


, for [x ] ¹ 0
30. If f ( x ) = ïí [x ] , where [x ] denotes the (c) - x + 2 x
2 3
(d) None of these
ï 0, for [x ] = 0
î 36. Let [ ] represents the greatest integer function less than
greatest integer less than or equal to x, then æ é n sin x ù é n tan x ù ö
or equal to x. The value of lim ç ê + ÷
lim f ( x ) equals
x ® 0
x ® 0 èë x úû êë x úû ø
(a) 1 (b) 0 is
(c) -1 (d) doesn’t exist (a) n + 1 (b) 2n
(c) n - 1 (d) 2n - 1
| x 3 – 6x 2 + 11x – 6 |
31. Let f ( x ) = , then the number of
37. The value of lim [ 2 – x + 1 + x ] , where a Î éê0, ùú and
1
x 3 – 6x 2 + 11x – 6
x ®a ë 2û
solutions of ‘a’, where lim f ( x ) doesn’t exist is
(a) 3 (b) 2 x®a
(c) 1 (d) 4 [ × ] denotes the greatest integer function is
(a) 1 (b) 2
32. Let the rth term, t r of a series is given by (c) 3 (d) 4
n
r
tr =
1+r2 +r 4
. The value of lim
n ®¥
åt r is 38. The value of
r =1 (tan x – sin x )
1 1
(a) 2 (b) (c) 1 (d) –1 + (tan x – sin x ) + (tan x - sin x )
2 4 +
+ . .. ¥
æ 3 1ö
n çr – r + ÷ lim
x®0 +
is
33. The value of lim
n ®¥
å cot –1
ç
ç 2
r ÷ is
÷
–1 + x 3 + x 3 + x 3 + ... ¥
r =1 1 1
è ø (a) (b)
2 4
p 1
(a) p (b) (c) (d) 1
4 8
p
(c) (d) p 39. The value of
2
cos 2 (1 – cos 2 (1 – cos 2 (1 ... cos 2 q ))... )
34. Let (tan a ) x + (sin a ) y = a and lim is
q ®0 æ p ( q + 4 – 2) ö
(a cosec a ) x + (cos a ) y = 1 be two variable straight sin çç ÷÷
lines, a being the parameter. Let P be the point of è q ø
intersection of the lines. In the limiting position, when (a) 2 (b) 2
a ® 0, the point of intersection of straight lines is (c)
1
(d)
1
(a) (2, - 1 ) (b) (2, 1 ) 2 2
(c) ( -2, 1 ) (d) ( -2, - 1 )
40. The value of lim an when an + 1 = 2 + an ,
35. The polynomial of least degree, such that n ®¥
1/x
æ x + f (x )ö 2 n = 1 , 2 , 3 , K is
lim ç1 + ÷ = e 2 is (a) 1 (b) 2
x ®0 è x2 ø (c) 3 (d) 4

Limits Exercise 2 : More than One Option Correct Type Questions


æp x + 1ö 1 - cos ( x 2 )
41. If lim 4 x ç - tan -1 ÷ = y + 4y + 5, then y can
2
42. lim is equal to
x ®¥ è4 x + 2ø x ®0 x 3 ( 4 x - 1)
be equal to 1
(a) ln 2 (b) ln2
(a) 1 (b) - 1 2
(c) - 4 (d) - 3 1 æe2 ö
(c) ln 4 (d) 1 - ln ç ÷
2 è4ø
Chap 05 Limits 289

43. If f ( x ) = e [cot x ] , where [y ] represents the greatest where [×] denotes the greatest integer less than or equal
integer less than or equal to y, then to x, then
1 (a) lim f ( x ) = sin 1
(a) lim f ( x ) = 1 (b) lim f ( x ) =
p +
p + e x ® 0-

2

2
(b) lim f ( x ) = 0
x ® 0+
1
(c) lim f ( x ) = (d) lim f ( x ) = 1 (c) limit does not exist at x = 0
p- e p-
x® x® (d) limit exists at x = 0
2 2
50. lim f ( x ) does not exist when
44. lim éêm
sin x ù x ®c
is equal to (where m Î I and [ × ] denotes
x ®0 ë x úû (a) f ( x ) = [[ x ]] - [2 x - 1 ] , c =3 (b) f ( x ) = [ x ] - x, c =1
greatest integer function) tan(sgn x )
(c) f ( x ) = { x } 2 - { - x } 2, c = 0 (d) f ( x ) = , c =0
(a) m, if m < 0 (sgn x )
(b) m - 1, if m > 0 (where [×] and {×} denotes greatest integer and fractional
part of x)
(c) m - 1, if m < 0
(d) m, if m > 0 51. Identify the correct statement.
é n 1ù
45. If lim (1 + ax + bx 2 ) 2 / x = e 3 , then (a) lim ê S r ú = 1
x ®0 n ®¥ ë r = 1 2 û

(a) a = 3, b = 0
3
(b) a = , b = 1 (b) If f ( x ) = ( x -1 ) { x }, where [×] and {×} denotes greatest integer
2 function and fractional part of x respectively, the limit of
3 f ( x ) does not exist at x =1
(c) a = , b = 4 (d) a = 2, b = 3
2 é tan x ù
(c) lim ê
x ®0 ë x û
+ ú =1
46. The graph of the function y = f ( x ) is shown in the
é tan x ù
adjacent figure, then correct statement is (d) ê lim =1
ë x ® 0 + x úû
Y
a cot x - a cos x
2 52. For a > 0, let l = lim and
x ® cot x - cos x
p
(–1, 1) 1 2

4 m = lim ( x 2 + ax - x 2 - ax ), then
X x ®-¥
–1 0 1 2
(a) l > m, for all a > 0 (b) l > m, only when a ³1
(c) l > m, for all a > e -a (d) el + m = 0
x=3
x
æ ax + 1 ö
53. Consider the function f ( x ) = ç ÷ , where a, b > 0 ,
(a) lim f ( x ) = 1 (b) lim f ( x ) = 2 è bx + 2 ø
x ® 0+ x ®1
(c) lim f ( x ) = does not exist (d) lim f ( x ) = 4 the lim f ( x ) is
x ®3 x ®4 x ®¥
(a) exists for all values of a and b
æ1ö (b) zero for a < b
cot -1 ç ÷
èx ø (c) non-existent for a > b
47. For lim
x ®0 x (d) e -(1/a ) or e -(1/b ) , if a = b
(a) RHL exists (b) LHL does not exist x ×2x - x æ log 2 ö
(c) limit does not exist as RHL is 1 and LHL is -1 54. If f ( x ) = and g ( x ) = 2 x × sin ç x ÷, then
1 - cos x è 2 ø
(d) limit does not exist as RHL and LHL both are non-existent
x (a) lim f ( x ) = log 2 (b) lim g( x ) = log 4
æ x + 1ö x ®0 x ®¥
48. If l = lim ç ÷ , the value of {l } and [l ] are
x ®¥ è x - 1 ø
(c) lim f ( x ) = log 4 (d) lim g( x ) = log 2
x ®0 x ®¥
(a) 7 (b) 7 -e 2 (c) -7 (d) e 2 - 7
x 3 + cx 2 + 5x + 12
ì sin [x ] 55. If lim = l (finite real number), then
,[x ] ¹ 0 x 2 - 7 x + 12
49. If f ( x ) = ïí [x ]
x ®3
,
ï0, (a) l = 4 (b) c = - 6
î [x ] = 0 (c) c = 4 (d) x Î R
Limits Exercise 3 : Passage Based Questions
1/n
Passage I (Q. Nos. 56 to 58) æ K1 ö
If lim f ( x ) = 1 and lim g ( x ) = ¥,then
62. The value of lim ç ÷ is
n ®¥ èK2 ø
x ®a x ®a
lim ( f ( x ) - 1) ´ g ( x ) 4 1
(a) e 2 (b) log 4 - 1 (c) (d)
lim { f ( x )} g (x ) = e x ® a . e e
x ®a
sin x
æ K1 ö
56. lim æç
sin x ö x - sin x 63. The value of lim ç ÷ is
÷ is equal to n ®¥ èK2 + K3 ø
x ®0 è x ø

(a) 1/e (b) - 1/e (a) e (b) 1


(c) e (d) - e (c) 0 (d) does not exist
1
æ x - 1 + cos x ö x Passage IV (Q. Nos. 64 to 65)
57. lim ç ÷ is equal to
x ®0 è x ø Let f : N ® R and g : N ® R be two functions and
1 f (1) = 0.8, g (1) = 06
. ; f (n + 1) = f (n ) cos( g (n )) - g (n ) sin( g (n ))
(a) e1/2 (b) e -1/2 (c) e1 (d) and g (n + 1) = f (n ) sin( g (n )) + g (n )cos( g (n )) for n ³ 1.
e
2
64. lim f (n ) is equal to
æa x + b x + c x ö x n ®¥
58. lim ç ÷ is equal to
x ®0 è 3 ø (a) -1 (b) 0
(c) 1 (d) does not exist
(a) a 2/3 + b 2/3 + c 2/3 (b) abc
(c) (abc ) 2/3 (d) 1 65. lim g (n ) is equal to
n ®¥
(a) -1 (b) 0
Passage II (Q. Nos. 59 to 61) (c) 1 (d) does not exist
n
æ xö
Let f ( x ) = lim ç cos ÷ , g ( x ) = lim (1 – x + x e ) .
n n
Passage V (Q. Nos. 66 to 68)
n ®¥ è nø n ®¥
A circular arc of radius 1 subtends an angle of x radian as shown
Now, consider the function y = h ( x ), where in figure. The centre of the circle is O and the point C is the
h ( x ) = tan - 1 ( g - 1 f - 1( x )). intersection of two tangent lines at A and B. Let T ( x ) be the area
ln ( f ( x )) of DABC and S( x ) be the area of shaded region.
59. lim is equal to C
x ®0 ln ( g ( x ))
1 1
(a) (b) - B
2 2
(c) 0 (d) 1
60. Domain of the function y = h( x ) is A
(a) ( 0, ¥ ) (b) R x
(c) ( 0, 1 ) (d) [ 0, 1 ] O
61. Range of the function y = h( x ) is T (x )
66. lim is
æ pö æ p ö x ®0 x3
(a) ç 0, ÷ (b) ç - , 0 ÷
è 2ø è 2 ø 1 1 1 1
(a) (b) (c) (d)
æ p pö 2 3 4 8
(c) R (d) ç - , ÷
è 2 2ø S( x )
67. lim is
x ®0 x
Passage III (Q. Nos. 62 to 63) 1
(a) 0 (b) (c) 1 (d) None of these
Let K 1 = Total number of ways of selecting of ball from a bag 2
which contains n balls of first colour, (n + 1) balls of second colour, 68. lim
T (x )
is
(n + 2) balls of third colour, …, (2n - 1) balls of n colour. x ® 0 S( x )
K 2 = number of n-digit numbers using the digits 1, 2, 3, …, n and 1 3 3
(a) (b) (c) (d) 0
K 3 = number of ways of arranging (n + 1) objects on a circle. 4 4 2
Chap 05 Limits 291

Limits Exercise 4 : Matching Type Questions


69. Match the statements of Column I with values of Column II. 71. Match the column.
Column I Column II Column I Column II
(A) lim tan -1 (tan x ) (p) 0 (A) 1 - cos 2 x (p) 1
p+
lim equals
x®0 x2

2 e -e +xx

é n 1ù æ (3 / x ) + 1ö
1/ x
(q) 2
lim ê å r ú ([ × ] denotes the
(B) (q) Doesn’t exist (B) If the value of lim ç ÷ can be
n® ¥ ê + è (3 / x ) - 1 ø
ë r = 1 2 úû x®0

greatest integer function) expressed in the form of e p / q , where p and


æ x ö (r) p q are relative prime, then ( p + q) is equal to
(C) lim sec- 1 ç ÷ -
x®¥ è x + 1ø 2 tan 3 x - tan x 3 (r) 4
(C) lim equals
cos x (s) p x®0 x5
(D) lim

p (1 - sin x )2/ 3 2 x + 2 sin x (s) 5
2 (D) lim
x®0 x + 2 sin x + 1 - sin 2 x - x + 1
2

70. A right angled triangle has legs 1 and x. The hypotenuse equals
is y and the angle opposite to the side x is q. Shown as
72. Match the column.
θ
Column I Column II
y
1
(A) lim cos (p (3 n + n + 2 n - n)),
2 3 2
(p) 1
n® ¥
2
where n is an integer, equals
x
Match the column. (B) lim n sin (2 p 1 + n2 ) (n Î N ) (q) 1
n® ¥
4
Column I Column II equals
(A) lim ( y - x ) (p) 0 (C) lim (- 1)n sin (p n2 + 0. 5n + 1) (r) p
p
q® n® ¥
2

(B) lim ( y - x ) (q) 1 æ (n + 1) p ö



p ç sin ÷ is (where n Î N )
2 2 è 4n ø
(C) lim ( y2 - x 2 ) (r) 1 x
p æ x + aö
q® (D) If lim ç ÷ = e, where a is some (s) non
x ® ¥ è x - aø
2
-existent
(D) lim ( y3 - x 3 ) (s) ¥ real constant, the value of a is equal to
p

2

Limits Exercise 5 : Single Integer Answer Type Questions


æ æ 1 öö 75. Let f (n ) = éê n + ùú, where [×] denotes greatest integer
1
73. Let L = lim ç x log x + 2x × log sin ç ÷ ÷, then the
è
x ®¥ è x øø ë 2û
æ 2ö function, " n Î N.
value of ç - ÷ is ………… .
è Lø ¥ - f (n )
(n + xn ) 2 f (n ) + 2
74. For n Î N , let x n be defined as æç1 + ö÷
1
= e , then
Then, å 2n
is equal to ……… .
è nø n =1
2 lim ( x n ) equals ………… .
n ®¥
292 Textbook of Differential Calculus

76. If the arithmetic mean of the product of all distinct pairs æ1ö 3
(3x 4 + 2x 2 ) × sin ç ÷ + x + 5
n 2 èx ø k
of positive integers whose sum is n, is S n , then lim 85. If lim 3 2
= k , then is
x ® -¥
n ® ¥ Sn x + x + x +1 2
must equal to ………… .
……........ .
¥
6n
77. If k = å , the value of k is ………… . é 2x æ x öù
(3n - 2n )(3n +1 - 2n +1 ) 86. If f ( x ) = lim ê × tan -1 ç 2 ÷ ú, then f (1) is ……...... .
n =1 t ®0 ë p èt øû
tan x – sin {tan –1 (tan x )} æ 1 ö
78. The value of lim is ……… . 1 1
p tan x + cos 2 (tan x ) 87. The value of lim ç + +¼+ ÷ is
n ®¥ ç
n 2 + 2n ÷ø

2 è n
2
n2 +1
79. The figure shows two regions in the first quadrant. A(t) 1

is the area under the curve y = sin x 2 from 0 to t and 1- x x sin ( x - [x ])


88. If l = lim+ 2 -2 and m = lim (where
B (t ) is the area of the triangle with vertices 0, P and x ®1 x -1x ®1+

A (t ) m [×] denotes greatest integer function). Then, (l +m ) is …… .


M (t , 0). If lim = , then (m + n ) is ………… .
t ® 0 B (t ) n é sin x × tan x ù
89. The value of lim ê ú is …….......
Y Y x ®0 ë x2 û
P(t, sin t2) P(t, sin t2)
(where [×] denotes greatest integer function).
x2

n kr
90. If lim S = 1, then k 2 is
sin

n ®¥ r =1 1 ´ 3 ´ 5 ´ ¼ ´ (2r - 1) ´ (2r + 1)
y=

A(t) B(t)
……...... .
X X
O t O M (t, 0) 1 1
91. If f ( x ) = lim sin 4 x + sin 4 2x + ¼ + × sin 4 (2n x ) and
æ 2t sin x ö ö
n ®¥ 4 4n
80. If the two lines AB : ç ò æç + 1÷ dx ÷ x + y = 3t and
è0 è x ø ø g ( x ) is a differentiable function satisfying g ( x ) + f ( x ) = 1,
AC : 2tx + y = 0 intersect at a point A, the x-coordinate of then the maximum value of ( f ( x ) + g ( x )) 4 is ……...... .
p
a point A as t ® 0, is equal to (p and q are in their 92. If f ( x + y + z ) = f ( x ) + f (y ) + f (z ) with f (1) = 1 and
q n
lowest form), the ( p + q ) is ………… . ( 4r ) f (3r )
f (2) = 2 and x, y, z ÎR, the value of lim S is ….
n ®¥ r =1 n3
x2
81. Consider a parabola y = and the point F(0, 1). Let n × 3n 1
4 93. If lim = , the number of the
n +1
A 1 ( x 1 , y 1 ), A 2 ( x 2 , y 2 ), A 3 ( x 3 , y 3 ), K, A n ( x n , y n ) are n ®¥ n × ( x
- 1) + n × 3 - 3
n 3 n

‘n’ points on the parabola such that x k > 0 and integral values of x is ............ .
kp
ÐOFA k = (k = 1, 2, K , n ) . If the value of 94. The value of
2n
lim ((( x - 1)( x - 2)( x + 3)( x + 10)( x + 15))1/ 5 - x ) is ...... .
1 n m x ®¥
lim × å FA k = , then m is ………… .
n ®¥ n p tan x
k =1 95. If lim ([ f ( x )] + x 2 ){ f ( x )} = k , where f ( x ) = and [×]
x ®¥ x
cos (tan -1 (tan x ))
82. If L = lim+ , then cos (2 p L ) is ……… . , {×} denotes greatest integer and fractional part of x

p x - p /2 respectively, the value of [k / e ] is ............. .
2
96. The value of lim {( 3 + 1) 2n } is ......... (where {×} denotes
83. Number of solutions of the equation cosecq = k n ®¥

n ær 3 - 1ö fractional part of x).


in [0, p], where k = lim P ç 3 ÷, is ………… .
97. If f ( x ) is a polynomial of degree 4, having extremum at
n ®¥ r = 2 è r + 1 ø
æ f (x )ö
84. If C satisfies the equation x = 1, 2 and lim ç1 + 2 ÷ = 2 .
x ® 0è x ø
x
æx +c ö ec Then, f (2) is ............ .
lim ç ÷ = 4, then is ……... .
x®¥ èx -c ø 2
Chap 05 Limits 293

98. If a is the number of solutions of | x | = log ( x - [x ]), 99. Suppose x 1 = tan -1 2 > x 2 > x 3 > ¼ are the real
(where [×] denotes greatest integer function) and numbers satisfying sin ( x n + 1 - x n ) + 2 - (n + 1) × sin x n
x e ax - b sin x × sin x n + 1 = 0 for all n > 1 and l = lim x n , the value of
lim is finite, the value of (a + b ) is n ®¥
x ®a x3 [4l ] is ........ (where [×] denotes greatest integer function).

Limits Exercise 6 : Questions Asked in Previous 10 Years’ Exams


(i) JEE Advanced & IIT-JEE
x 2 sin (bx )
100. Let a , b Î R be such that lim = 1. Then, F(x ) 1 æ1ö
x ® 0 ax - sin x If lim = , then the value of f ç ÷ is
x ® 1 G( x ) 14 è2ø
6 (a + b ) equals [Integer Answer Type, 2016 Adv]
[Integer Answer Type, 2015 Adv]
101. Let m and n be two positive integers greater than 1. If 103. The largest value of the non-negative integer a for
æ e cos (an ) - e ö 1- x
lim ç ÷ = - æç e ö÷ , then the value of m is ì - ax + sin( x - 1) + a ü 1- x 1
a®0 ç a m ÷ è2ø n which lim í ý = , is
è ø x ®1 î x + sin( x - 1) - 1 þ 4
[Integer Answer Type, 2015 Adv]
[Integer Answer Type , 2014 Adv]
102. Let f : R ® R be a continuous odd function, which
x2
1
vanishes exactly at one point and f (1) = . Suppose that a - a2 - x 2 -
2 104. Let L = lim 4 , a > 0 . If L is finite, then
x x ®0 x4
F ( x ) = ò f (t )dt for all x Î [-1 , 2] [More than One Correct Option, 2009]
–1
x (a) a = 2 (b) a = 1
and G( x ) = ò t | f { f (t )}| dt for all x Î [-1 , 2]. 1 1
–1
(c) L = (d) L =
64 32

(ii) JEE Main & AIEEE


cot x - cos x æx 2 + x + 1 ö
105. The lim equals 110. If lim ç - ax - b ÷ = 4, then
p ( p - 2x ) 3
x® [2017 JEE Main] x ®¥ è x +1 ø [2012 AIEEE]
2

(a)
1
(b)
1
(c)
1
(d)
1 (a) a = 1, b = 4 (b) a = 1, b = - 4
4 24 16 8 (c) a = 2, b = - 3 (d) a = 2, b = 3
106. Let p = lim+ (1 + tan 2 x )1/ 2 x , then log p is equal to 111. Let a (a ) and b (a ) be the roots of the equation
x ®0
[2016 JEE Main]
1 1 ( 3 1 + a - 1) x 2 - ( 1 + a - 1) x + ( 6 1 + a - 1) = 0, where
(a) 2 (b) 1 (c) (d)
2 4
a > - 1. Then, lim a (a ) and lim b (a ) are
1/n a ® 0+ a ®0 +
é (n + 1)(n + 2) K 3n ù [2012 AIEEE]
107. lim ê ú is equal to 5 1
n ®¥ ë n 2n û [2016 JEE Main] (a) - and 1 (b) - and -1
2 2
18 27 9
(a) 4 (b) 2 (c) 2 (d) 3 log 3 - 2 7 9
e e e (c) - and 2 (d) - and 3
2 2
sin( p cos 2 x ) 1
108. lim 2
is equal to 112. If lim [1 + x log (1 + b 2 )] x = 2b sin 2 q, b > 0
x®0 x [2014 JEE Main] x ®0
p
(a) (b) 1 (c) -p (d) p and q Î ( - p, p ], then the value of q is [2011 AIEEE]
2 p p
(a) ± (b) ±
(1 – cos 2x )(3 + cos x ) 4 3
109. lim is equal to
[2013 JEE Main] p p
x ®0 x tan 4 x (c) ± (d) ±
1 6 2
(a) 4 (b) 3 (c) 2 (d)
2
294 Textbook of Differential Calculus

é sin x ù (cos x - 1) (cos x - e x )


113. For x > 0, lim ê(sin x )1/ x + æç ö÷
1 116. The integer n for which lim
ú is is a
x ®0
ê èx ø úû
x ®0 xn
ë [2006 AIEEE]
finite non-zero number, is [2002 AIEEE]
(a) 0 (b) – 1 (c) 1 (d) 2
(a) 1 (b) 2 (c) 3 (d) 4
f (2h + 2 + h 2 ) - f (2) 117. Let f : R ® R be such that f (1) = 3 and f ¢ (1) = 6. Then,
114. lim , given that f ¢ (2) = 6 and
h ®0 f (h - h 2 + 1) - f (1) 1/ x
é f (1 + x ) ù
f ¢ (1) = 4, [2003 AIEEE] lim ê
x ® 0 ë f (1) û
ú equals [2002 AIEEE]
(a) does not exist (b) is equal to -3/2 1
(c) is equal to 3/2 (d) is equal to 3
(a) 1 (b) e 2 (c) e 2 (d) e 3
{(a - n ) nx - tan x } sin nx x
115. If lim = 0, where n is æx - 3 ö
x ®0 x2 118. For x Î R , lim ç ÷ is equal to
x ®¥ èx + 2 ø
non-zero real number, then a is equal to [2003 AIEEE] [2000 AIEEE]
-1
(a) e (b) e
n+1 1 (c) e -5 (d) e 5
(a) 0 (b) (c) n (d) n +
n n

Answers
Exercise for Session 1 Exercise for Session 4
1. (c) 2. (c) 3. (a) 4. (c) 5. (d) 6. (a) 1. (1) 2. (1) 3. (e) 4. (c)
7. (a) 8. (d) 9. (a)
Exercise for Session 5
Exercise for Session 2 1. (a) 2. (c) 3. (c) 4. (a)
1. (a) 2. (b) 3. (a) 4. (a) 5. (b)
Exercise for Session 6
Exercise for Session 3 1. (c) 2. (c) 3. (a) 4. (b) 5. (b)
1. (a) 2. (b) 3. (b) 4. (d) 5. (b) 6. (a)
7. (a) 8. (a) 9. (d) 10. (c)

Chapter Exercises
1. (a) 2. (c) 3. (a) 4. (c) 5. (c) 6. (d) 7. (c) 8. (c) 9. (b) 10. (c)
11. (d) 12. (d) 13. (d) 14. (d) 15. (a) 16. (a) 17. (c) 18. (c) 19. (b) 20. (b)
21. (c) 22. (c) 23. (a) 24. (c) 25. (c) 26. (a) 27. (a) 28. (c) 29. (b) 30. (d)
31. (a) 32. (b) 33. (c) 34. (a) 35. (c) 36. (d) 37. (b) 38. (a) 39. (b) 40. (b)
41. (b, d) 42. (b, d) 43. (b, d) 44. (a, b) 45. (b, c) 46. (a, b, c) 47. (a, b) 48. (a, d)
49. (a,b) 50. (b, c) 51. (c, d) 52. (b, c, d) 53. (b, c,d) 54. (c,d) 55. (a, b) 56. (a)
57. (b) 58. (c) 59. (b) 60. (c) 61. (d) 62. (c) 63. (b) 64. (a) 65. (b) 66. (d)
67. (a) 68. (c)
69. (A) ® (r); (B) ® (p); (C) ® (q); (D) ® (q)
70. (A) ® (p); (B) ® (p); (C) ® (r); (D) ® (s)
71. (A) ® (r); (B) ® (s); (C) ® (p); (D) ® (q)
72. (A) ® (q); (B) ® (r); (C) ® (p); (D) ® (p)
73. (6) 74. (1) 75. (3) 76. (6) 77. (2) 78. (1) 79. (5) 80. (5) 81. (4) 82. (1)
83. (0) 84. (1) 85. (1) 86. (1) 87. (2) 88. (2) 89. (1) 90. (4) 91. (4) 92. (4)
93. (3) 94. (5) 95. (7) 96. (1) 97. (0) 98. (1) 99. (3) 100. (7) 101. (2) 102. (7)
103. (2) 104. (c) 105. (c) 106. (b) 107. (d) 108. (c) 109. (b) 110. (d) 111. (b) 112. (d)
113. (c) 114. (d) 115. (d) 116. (c) 117. (c) 118. (c)

You might also like